Você está na página 1de 52

PROF ANDRÉ FONSECA GEOMETRIA

OS ELEMENTOS BÁSICOS DA GEOMETRIA


Os elementos básicos do estudo da 
Geometria são ideias de ponto, reta e plano. No nosso plano 
dia-a-dia usamos essas palavras em diversas
ocasiões, e com diversos significados, tais como: A Semi-reta
– Esse é o ponto de partida para a execução do Se pudéssemos cortar uma reta ao meio e
projeto. ficar apenas com a sua metade, teríamos o que
– A que ponto chegamos! chamamos de semi-reta e o ponto onde a reta foi
cortada seria o ponto de origem da semi-reta.
– Estamos na reta final do trabalho. Graficamente, uma semi-reta pode ser
– Eu tenho um plano! representada pela figura a seguir:
A
Sob o ponto de vista da Geometria, no O
entanto, essas palavras têm significados muito
específicos. Contudo, apesar de serem conceitos semi-reta OA
importantes, são difíceis de serem definidos por serem
intuitivos. O Segmento de Reta
Tente dar uma definição de um deles: Vamos considerar a figura a seguir que representa
– O que é reta? uma reta que contém os pontos A e B.
A B

O PONTO, A RETA E O PLANO


A parte da figura que fica entre os pontos A e B,
O ponto, a reta e o plano não existem no incluindo os pontos A e B, é o que chamamos de
mundo real: um grão de areia, uma vareta ou um segmento de reta. Neste caso, A e B são chamados
tampo de mesa nos dão uma ideia de ponto, de reta e extremidades do segmento AB.
de plano. Mas nunca vimos um grão que não tenha
volume (por menor que ele seja), uma vareta que não Para trabalhar com esses conceitos,
tenha espessura e se prolongue indefinidamente, ou precisamos raciocinar de forma a evitar erros.
um tampo de mesa que se prolongue em todas as Queremos encontrar propriedades que sejam
direções. verdadeiras.
Justamente porque ponto, reta e plano não
O ponto existem no mundo real, é importante que usemos
Graficamente, um ponto pode ser certas regras que permitam dizer se nossas
representado pela figura “”, e é indicado por letras conclusões são verdadeiras ou não. Nem sempre os
maiúsculas do nosso alfabeto. Assim, temos: nossos sentidos, ou o nosso bom senso, nos levam a
conclusões válidas, como você verá nos exemplos a
seguir.
A reta
Uma reta é uma figura com infinitos pontos. Na figura 1 a seguir, com o auxílio de uma
Graficamente, uma reta pode ser representada pela régua, veja se as linhas que ligam M a N e P a Q são
figura a seguir, e é indicada por letras minúsculas do linhas retas.
nosso alfabeto. u

reta u
M N
Podemos também indicar uma reta pelos
pontos que pertencem a ela. Por exemplo: se uma P Q
reta contém os pontos A e B, podemos indicá-la por:
reta AB ou simplesmente reta AB, quando especificado
que AB é uma reta.
Fig. 1
Quando três ou mais pontos pertencem à
mesma reta, eles são chamados de pontos colineares. Na figura 2 abaixo, qual das linhas é maior: a
horizontal ou a vertical?

O plano
Graficamente, um plano pode ser
representado pela figura a seguir (um paralelogramo),
e é indicado por uma letra minúscula do alfabeto
grego:  (alfa),  (beta),  (gama),  (delta) etc.
1
D

A ponto D
PROF ANDRÉ FONSECA GEOMETRIA
ponto A

Fig. 2: Qual é a maior linha?

indicação  u  v uv=P
Bem, se por um lado não podemos confiar OBS.: Há um caso particular de retas concorrentes
apenas no bom senso e na intuição, por outro, eles que são as retas que se cruzam formando quatro
são muito importantes. Isto porque em Geometria, ângulos congruentes (iguais); a representação deste
algumas afirmações são aceitas como sendo caso lembra a figura do sinal de mais (+).
verdadeiras sem quaisquer contestações, pois são u
situações bastante intuitivas.
Veja a seguir algumas dessas afirmações. v
representação 
 Por um único ponto passam inúmeras retas.
 Por dois pontos distintos (ou seja, diferentes),
passa uma única reta. indicação  uv

Medida de um segmento de reta


Determinar a medida de um segmento de reta é medir Retas Coincidentes
o seu comprimento, ou seja, dizer qual é o seu Duas retas são ditas coincidentes quando têm
tamanho. Para isso, precisamos de uma unidade de mais de um ponto em comum.
medida, que é o que tomaremos por comparação
representação  v
(medir quer dizer comparar), como por exemplo: o uv=u
palmo, o passo, a jarda, o metro, o quilômetro etc. u
ou
Observe a figura a seguir.
indicação  uv uv=v
X Y

Retas Reversas
Duas retas são ditas reversas quando não
possuem pontos em comum e se encontram em
planos diferentes. Observe. v

u
P Q B
A u
Tomando como unidade de medida o segmento u,
você seria capaz de dizer as medidas dos segmentos:
XY  No desenho acima, temos um cubo, uma
PB  figura com várias faces. As retas u e v estão em faces
PQ  diferentes e não se cruzam.

POSIÇÕES RELATIVAS DE DUAS RETAS


Num mesmo plano, duas retas podem ser: EXERCÍCIOS
paralelas, concorrentes ou coincidentes. E, quando
em planos diferentes, podem ser, também, reversas. 1. Diga se cada uma das afirmações abaixo é verdadeira ou
Veja. falsa.
 Por um ponto passam infinitas retas.
 Por três pontos dados passa uma reta.
Retas paralelas
 Quatro pontos dados, todos distintos, determinam
Duas retas são ditas paralelas quando não
duas retas.
têm pontos em comum.
u  Se dois pontos distintos A e B pertencem às retas r e
v s, então r = s.
representação   Duas retas distintas que têm um ponto em comum
são concorrentes.
uv=  Quatro pontos distintos, sendo três deles colineares,
determinam quatro retas.
indicação  u // v
2. Dados três pontos distintos de uma reta, quantos
Retas Concorrentes (ou secantes) segmentos distintos eles determinam?
Duas retas são ditas concorrentes (ou
secantes) quando têm apenas um ponto em comum. 3. Marque numa folha quatro pontos distintos, três a três
não colineares. Quantas retas podemos traçar passando por
u dois desses pontos?
representação  v
P
2
PROF ANDRÉ FONSECA GEOMETRIA

4. Dados dois pontos distintos, A e B, quantos segmentos


há com extremidades A e B? Quantos segmentos há que
passam pelos pontos A e B?

5. Faça um desenho onde constem os pontos A, B, C, D e


E, e retas r e s, satisfazendo ao mesmo tempo os itens a
seguir:
 r e s não são coincidentes;
 A  r e A  s;
 B  r e C  r;
 B e C estão em semiplanos opostos com respeito a s;
 D e E estão em semiplanos opostos com respeito a r,
e nenhum dos dois pontos pertence a s.

6. Desenhe dois segmentos AB e CD tais que a interseção


de AB e CD é o conjunto vazio, mas AB e CD têm um ponto
em comum.

7. Desenhe dois segmentos AB e CD tais que a interseção


de AB e CD é o conjunto vazio, mas AB = CD.

8. Escreva o que significa dizer que três pontos não são


colineares.

9. (ESA) Na figura abaixo, o segmento AB mede 14 cm e o


segmento MN mede 12 cm, M é o ponto médio de AB e N o
ponto médio de BC. A medida do segmento AC, em cm, é:
A M B N C

a) 28 b) 20 c) 12 d) 19 e) 24

10. (ESA) Considere os pontos colineares A, B, O e C na


ordem OABC. Se AO = 3 cm, OB = 5 cm e 4AB + AC – 2BC
= 6, então a distância, em cm, entre os pontos O e C é igual
a:
a) 5 b) 6 c) 7 d) 8 e) 9

s
11. Observe a figura e leia r
com atenção as afirmações
que seguem a seu respeito.
u
I. As retas t e u são
concorrentes oblíquas. t
II. As retas s e u são
reversas.
III. As retas s e t são concorrentes perpendiculares.
IV. As retas u e r são paralelas.
V. As retas t e s são coplanares.

De acordo com a figura anterior, a alternativa correta é:


a) todas as afirmações acima são verdadeiras
b) nenhuma das afirmações acima é verdadeira
c) As falsas são I, IV e V
d) as verdadeiras são II, III IV
e) as verdadeiras são II e IV

3
PROF ANDRÉ FONSECA GEOMETRIA

a) Um jogador ser carregado para casa pela orelha para fazer


lição.

b) Jogador que arrancou o tampão do dedão do pé. Porém,


UNIDADES DE MEDIDA DE nestes casos, o mesmo acaba voltando a partida após utilizar
aquela água “santa” da torneira do quintal de alguém.

As 10 regras do Futebol de Rua, c) Em caso de atropelamento.


o verdadeiro futebol de macho! 9. AS PENALIDADES
A única falta prevista nas regras do futebol de rua é atirar o
1. A BOLA adversário dentro do bueiro.
A bola pode ser qualquer coisa remotamente esférica. Até
uma bola de futebol serve. No desespero, usa-se qualquer 10. A JUSTIÇA ESPORTIVA
coisa que role, como uma pedra, uma lata vazia ou a Os casos de litígio serão resolvidos na porrada, prevalece os
merendeira do irmão menor. mais fortes e/ou quem pegar uma pedra antes.
2. O “GOL” QUEM NÃO JOGOU, PERDEU UM DOS MELHORES
O gol pode ser feito com o que estiver à mão: tijolos, MOMENTOS DA VIDA.
paralelepípedos de concreto, camisas emboladas, chinelos,
os livros da escola e até o seu irmão menor. Se for golzinho, Extraído e adaptado do site
há uma distância de mais ou menos 4 ou passos, dependo do http://estadio97.uol.com.br/forumMsg.asp?forumId=3821&vis=1
tamanho do pé (38, 40 etc). Acessado em 17 de março de 2011.

3. O CAMPO O texto acima, escrito por um jovem chamado


O campo pode ser só até o fio da calçada, calçada e rua, rua Carlos Eli nos leva a um momento incrível de nossas
e a calçada do outro lado e, nos grandes clássicos, o vidas (pelo menos para os homens).
quarteirão inteiro. Há um momento em que, ao falar sobre o
“gol”, é citada a distância entre os objetos utilizados –
4. DURAÇÃO DO JOGO 4 ou 5 passos (aqui, o passo se refere ao
O jogo normalmente vira 5 e termina 10; pode durar até a comprimento do pé do jogador).
mãe do dono da bola chamar ou escurecer. Nos jogos Neste caso, “o passo” é a unidade de medida
noturnos, até alguém da vizinhança ameaçar chamar a utilizada para medir a distância entre os objetos.
polícia. Saiba que medir significa comparar.
Portanto, para medir uma distância, é
5. FORMAÇÃO DOS TIMES necessário ter alguma coisa que se possa comparar
Varia de 3 a 70 jogadores de cada lado. Ruim vai para o gol. com essa distância.
Perneta joga na ponta, na esquerda ou na direita, No caso do “golzinho”, a distância está entre
dependendo da perna que faltar. De óculos é meia-armador, Para medir uma distância, é preciso uma unidade de
para evitar os choques. Os com mais corpo é beque. medida. Esta unidade é o que será comparada com a
distância em questão.
6. O JUIZ
Não tem juiz. uma baliza e outra e a unidade de medida utilizada foi
o passo.
7. AS INTERRUPÇÕES
No futebol de rua, a partida só pode ser paralisada em 3
eventualidades: AS DIFERENTES UNIDADES DE MEDIDA
AO LONGO DA HISTÓRIA
a) Se a bola entrar por uma janela. Neste caso os jogadores
devem esperar 10 min pela devolução voluntária da bola. Se
isso não ocorrer, os jogadores devem designar voluntários
para bater na porta da casa e solicitar a devolução, primeiro
com bons modos e depois com ameaças de depredação.
Medidas de comprimento:
b) Quando passar na rua qualquer garota gostosa. Côvado, do cotovelo à ponta dos dedos = 45
centímetros
c) Quando passarem veículos pesados, de ônibus para cima.
Bicicletas e Fusquinhas podem ser chutados junto com a
Braça, 4 côvados = 1.80 metros
bola e, se entrar, é gol. Estádio, 400 côvados = 1.480 metros
Milha = cerca de 3 metros
8. AS SUBSTITUIÇÕES Caminho de um sábado = aproximadamente
São permitidas substituições nos casos de: 1.080 metros
4
PROF ANDRÉ FONSECA GEOMETRIA

s elementos básicos do estudo da Geometria


são ideias de ponto, reta e plano. No nosso dia-a-dia
usamos essas palavras em diversas ocasiões, e com
diversos significados, tais como:
– Esse é o ponto de partida para a execução do
projeto.
– A que ponto chegamos!
– Estamos na reta final do trabalho.
– Eu tenho um plano!
Sob o ponto de vista da Geometria, no
entanto, essas palavras têm significados muito
específicos. Contudo, apesar de serem conceitos
importantes, são difíceis de serem definidos por serem
intuitivos.
Tente dar uma definição de um deles:
– O que é reta?

O PONTO, A RETA E O PLANO

O ponto, a reta e o plano não existem no


mundo real: um grão de areia, uma vareta ou um
tampo de mesa nos dão uma ideia de ponto, de reta e
de plano. Mas nunca vimos um grão que não tenha
volume (por menor que ele seja), uma vareta que não
tenha espessura e se prolongue indefinidamente, ou
um tampo de mesa que se prolongue em todas as
direções.

O ponto
Graficamente, um ponto pode ser
representado pela figura “”, e é indicado por letras
maiúsculas do nosso alfabeto. Assim, temos:

5
ÂNGULOS

PROF ANDRÉ FONSECA GEOMETRIA

Ângulo obtuso:
Ângulo cuja medida está entre 90º e 180º.
Ângulos são figuras geométricas formadas por
duas semi-retas de mesma origem.
B Ângulo obtuso AÔB
O A
Para a figura ao lado,damos a
indicação AÔB ou apenas Ô.
As semi-retas OA e OB são os A O
lados do ângulo.
B
Dizemos ainda que o ponto O é Ângulo raso ou meia volta:
o vértice do ângulo. Ângulo que tem medida igual a 180º.
A cada ângulo podemos associar trabalhar com taisou seja,
um número,
uma medida. A unidade medidas de medidaicialmente,
que vamos,
utilizarcom
inicialmente, utilizar para trabalhar paratais
trabalhar
medidas é Ângulo raso AÔB
o Grau.
A O B
Tomando uma circunferência e dividindo-a em 360
partes iguais, cada um dos ângulos centrais obtidos Ângulo rombo ou uma volta:
por essa divisão tem como medida 1 grau (indica-se: Ângulo que tem medida igual a 360º.
1º).
A
Ângulo de uma volta AÔB
O B

OUTRAS CLASSIFICAÇÕES...

Ângulos congruentes:
Dois ângulos são congruentes quando possuem e
mesma medida.
A C
O
D

CLASSIFICAÇÃO PARA OS ÂNGULOS DE B


ACORDO COM SUAS MEDIDAS: m(AÔB) = 35º O
m(CÔD) = 35º
Ângulo nulo:
É a figura formada por duas semi-retas coincidentes,
Ângulos adjacentes:
considerando que não há abertura entre elas.
Dois ângulos são adjacentes quando têm o mesmo
O A vértice e um dos lados em comum e não possuem
Ângulo nulo AÔB
B pontos interiores em comum.
Ângulos AÔB e BÔC são
Ângulo agudo: B
A adjacentes e possuem o
Ângulo cuja medida está entre 0º e 90º. lado OB em comum
B C
Ângulo agudo AÔB menor O
que 90º e maior que 0º

O A Ângulos complementares:
Dois ângulos são complementares quando a soma de
Ângulo reto:
suas medidas é 90º.
Ângulo que tem medida igual a 90º.
B
Ângulo reto AÔB

O A
PROF ANDRÉ FONSECA GEOMETRIA

Na figura, o ângulo AÔB tem medida b


e o ângulo BÔC tem medida a. Como 02. (ENEM-2004) Nos X-Games Brasil, em maio de
C B os ângulos BÔC e AÔB juntos formam 2004, o skatista brasileiro Sandro Dias, apelidado
a um ângulo reto (a + b = 90º) então, “Mineirinho”, conseguiu realizar a manobra
eles são complementares.
b Dizemos ainda que o complemento do
denominada “900”, na modalidade skate vertical,
O ângulo a é 90º - a e o complemento do tornando-se o segundo atleta no mundo a conseguir
A
ângulo b é 90º - b. esse feito. A denominação “900” refere-se ao número
de graus que o atleta gira no ar em torno de seu
próprio corpo, que, no caso, corresponde a
a) uma volta completa.
Ângulos suplementares:
b) uma volta e meia.
Dois ângulos são suplementares quando a soma de
c) duas voltas completas.
suas medidas é 180º.
Na figura, AÔB e BÔC, de d) duas voltas e meia.
medidas, respectivamente a e b,
são suplementares, pois 03. A medida do suplemento de um ângulo cuja
B formam, juntos, um ângulo de medida é a, é:
a meia volta.
b Dizemos ainda que o a) 90º – a c) 90 + a
A O C
suplemento do ângulo a é (180º b) 180º – a d) 180º + a
- a) e que o suplemento do
ângulo b é (180º - b). 04. (U. Passo Fundo – RS) A diferença entre dois
ângulos suplementares é 48º. O maior deles mede:
Bissetriz de um ângulo a) 42º b) 69º c) 76º d) 204 e) 114º
Bissetriz de um ângulo é a semi-reta com origem no
vértice desse ângulo e que o divide em dois outros 05. Na figura abaixo, a medida de x é:
ângulos congruentes (iguais).

A Na figura, a medida do ângulo C B


C
CÔA é igual a medida do ângulo
AÔD, portanto, OA é a bissetriz do 2x + 20º
D ângulo CÔD. x + 10º
O A

O a) 20º b) 45º c) 27º d) 9º


Ângulos opostos pelo vértice 06. Na figura abaixo, a medida do menor ângulo é:
Dois ângulos são chamados opostos pelo vértice
quando os lados de um ângulo são semi-retas opostas
aos lados do outro. 2x + 20º
Dois ângulos opostos pelo vértice são sempre 3x – 65º
congruentes, ou seja, têm a mesma medida.
A O B
Na figura, AÔB e CÔD são
C opostos pelo vértice O.
a) 45º b) 70º c) 20º d)110º

O A 07. Na figura abaixo, qual o valor de x?


D
2x - 30º
x
 30º
B 2

O
EXERCÍCIOS
08. Na figura abaixo, qual o valor de x e y?
01. Descubra a medida dos ângulos centrais
desconhecidos.
50º 2x + 20º
30º y
60º
50º 80º
?
09. Sobre a figura abaixo,
40º C é correto afirmar que:
100º 40º
? 50º D B

x x
E x x
x A
7 O x
xx

F H
G
PROF ANDRÉ FONSECA GEOMETRIA

a) AÔB = AÔC c) AÔC = EÔG


b) AÔC = BÔE d) BÔD = CÔF

RETAS PARALELAS CORTADAS POR


10. Qual o menor ângulo formado pelos ponteiros de UMA TRANSVERSAL
um relógio, supondo um relógio analógico, quando o
relógio estiver marcando:
a) 2h Duas retas paralelas r e s, interceptadas por uma
b) 18h transversal, determinam oito ângulos, assim
c) 21h denominados.
d) 12:30h
e) 3:40h b
a r
c d
11. Qual é o ângulo que somado ao triplo do seu
complemento é igual a 210? f e
s
12. Determine dois ângulos suplementares, sabendo g h
que um deles é o triplo do outro.

13. Qual é a medida do ângulo formado pelas Os ângulos formados têm nomes particulares. Veja:
bissetrizes de dois ângulos adjacentes e
suplementares? Ângulos correspondentes: a e e, b e f, c e g, d e h;
Ângulos alternos internos: c e e, d e f;
Ângulos alternos externos: a e g, b e h;
GABARITO: Ângulos colaterais internos: c e f, d e e; e
1- a) 110º b) 160º; 2-D; 3-B; 4-E; 5-A; 6-B; 7-40º; 8-x=15º e y=130º;
9-C; 10- a) 60º, b) 180º, c) 90º, d) 165º, e) 130º; 11-30º; 12-45º e
Ângulos colaterais externos: a e h, b e g.
135º, 13-90º;
PROPRIEDADES
 Ângulos alternos internos são
 Ângulos alternos externos são congruentes
 Ângulos correspondentes são

 Ângulos colaterais internos são


suplementares
 Ângulos colaterais externos são

EXERCÍCIOS
01. Na figura, r // s e t é transversal.
t
a d
r
b c
e h
s
f g

Então, a afirmativa correta é:


a) a=b
b) b + h = 180º
c) c + h = 180º
d) b=e

02. Dadas as retas paralelas cortadas por transversal


abaixo, faça o que se pede:

8
PROF ANDRÉ FONSECA GEOMETRIA
112º
a) Calcule x, y e z, sabendo que 2x + y + z = 240º. x
y s

x
09. Duas retas paralelas cortadas por uma transversal
formam ângulos alternos externos expressos em
z
graus por 13x – 8º e 6x + 13º.
A medida desses ângulos vale:
a) 31º b) 3º ou 177º c) 30º e 150º d) 62º e) 93º
b) Calcule x e y.
10. Para calcular a circunferência terrestre, o sábio
Erastóstenes valeu-se da distância conhecida de
r 800km entre as localidades de Alexandria e Siena no
2x 4x Egito (A e B, respectivamente), situadas no mesmo
y meridiano terrestre. Ele sabia que quando em Siena
s
os raios solares
120º caiam verticalmente, em Alexandria eles faziam um
ângulo de 7,2º com a vertical. Calcule, com esses
dados, a circunferência terrestre, isto é, o
Então, a afirmativa correta é: comprimento de uma volta completa em torno da
e) a=b Terra.
f) b + h = 180º
g) c + h = 180º
7,2º raios
h) b=e A solares

800km
03. Qual é o menor ângulo formado pelos ponteiros de B
um relógio, supondo o relógio analógico, quando este
marcar 2:30h? E quando marcar 2:45h?

04. Qual é o ângulo que somado ao triplo do seu


complemento é igual a 210º?
11. (UNIRIO) As retas r e s são paralelas. O valor do
05. Determine dois ângulos suplementares, sabendo ângulo  apresentado na figura abaixo é:
que um deles é o triplo do outro.

06. Qual é a medida do ângulo formado pelas 


bissetrizes de dois ângulos adjacentes e
suplementares?
130º
07. Na figura, AÊJ = DÔI e CÔI é o complemento de
DÔI. Identifique a sentença correta.
C D
A B F
a) 40º b) 50º c) 130º d) 45º e) 65º
E
12. (CAP-UFRJ-06) Na figura a seguir, considere o
par de retas t, v e o par de retas r, s.
O I
G
K
J

a) AF // GI.
b) CK  GI.
c) JÔK é o complemento de DÊF.
d) DÔI é suplemento de IÔK.
a) Indique o par de retas paralelas.
b) De acordo com a orientação apresentada,
08. Calcule o valor de x, sendo r//s. determine se o outro par de retas se interceptará ao
r
norte, ao sul, à leste ou à oeste. Justifique.
40º

9
PROF ANDRÉ FONSECA GEOMETRIA

 obtusângulo: se, e somente se, tem um ângulo


GABARITO obtuso.
1-C; 2-A; 3-40.000km; 4-72º; 5-A; 6-B; 7-A; 8-a) t e v, b)?; 9-

É possível desenhar um triângulo


TRIÂNGULOS acutângulo escaleno e um triângulo
obtusângulo isósceles? E triângulos
equiláteros obtusângulos? Procure
1. CONCEITO verificar quais as combinações possíveis
Os triângulos, assim como as retas, os ângulos, de acordo com seus desenhos.
os segmentos de reta etc. são objetos idealizados,
nascidos da observação de objetos materiais com
forma triangular (como por exemplo um guardanapo
de papel dobrado). Veja se você consegue identificar OBS.:
alguns triângulos na figura a seguir. O lado oposto ao ângulo reto de um triângulo retângulo é chamado de
hipotenusa (maior lado) e os outros são chamados de catetos.
Condição de Existência
1.1. Definição
1.4. “Em todo triângulo, a medida de qualquer lado é
Dados três pontos A, B e C não colineares, à
menor que a soma das medidas dos outros lados.”
reunião dos segmentos AB, AC e BC chama-se
Chamando as medidas dos três lados de um triângulo
triângulo ABC (ou ΔABC). A qualquer de a, b e c, temos:

a<b+c b
Indicação: b<a+c a
ABC = AB  AC  BC c<a+b
B C
c
Exemplo
É possível formar um triângulo com as medidas
1.2. Elementos de um Triângulo
2cm, 3cm e 6cm?
No ABC acima, temos:
Vejamos...
lados: AB, BC e AC Se cada medida deve ser menor que a soma das
vértices: A, B e C outras, então...
ângulos internos: , e 2 < 3 + 6 => verdadeiro
ou ainda, , e . 3 < 2 + 6 => verdadeiro
6 < 2 + 3 => falso
1.3. Classificação Conclusão
» Não é possível formar um triângulo com essas medidas.
1.3.1. Quanto aos lados
Quanto aos lados, os triângulos podem ser
classificados da seguinte maneira: 2. PROPRIEDADES QUE RELACIONAM OS
 equiláteros: se, e somente se, têm os três lados ÂNGULOS DE UM TRIÂNGULO
congruentes;
 isósceles: se têm dois lados congruentes; e 1ª propriedade:
A soma dos ângulos internos de um triângulo
 escalenos: se, e somente se, dois quaisquer lados qualquer é igual a 180º.
não são iguais, ou seja, todos os lados diferentes. A
1.3.2. Quanto aos ângulos
Quanto aos ângulos, os triângulos podem ser
classificados da seguinte maneira:
 retângulo: se, e somente se, têm um ângulo reto; B C
 acutângulo: se, e somente se, têm os três ângulos
agudos; e

OBS.:
10
1. Um triângulo com dois lados congruentes é isósceles; o outro lado é
chamado de base do triângulo isósceles.
2. Todo triângulo equilátero é também isósceles.
PROF ANDRÉ FONSECA GEOMETRIA

3.2. Bissetriz (interna)


2ª propriedade: Segmento que une um vértice a um ponto qualquer do
A medida do ângulo externo de um triângulo lado oposto a ele e divide o ângulo desse vértice em
qualquer é igual à soma das medidas dos ângulos dois ângulos de mesma medida (congruentes).
internos não-adjacentes a ele. Todo triângulo possui três bissetrizes, que se
A encontram em um ponto chamado Incentro.

A
x̂ No triângulo ao lado temos:
B AD é a bissetriz relativa ao ângulo Â;
C BE é a bissetriz relativa ao ângulo B;
F E
I CF é a bissetriz relativa ao ângulo C.
Na figura acima, x é um ângulo externo não adjacente
aos ângulos A e C.
Pela 2ª propriedade, temos: B C
D
Incentro

3.2.1. Propriedade do Incentro


3ª propriedade: 3.2.1.1. Em todo triângulo, o Incentro é
Num triângulo, ao maior lado opõe-se o maior equidistante (tem a mesma distância) dos três lados.
ângulo, e ao menor lado opõe-se o menor ângulo. 3.2.1.2. O incentro é, também, o centro do
círculo inscrito no triângulo.

3. PONTOS NOTÁVEIS I

3.1. Mediana
Segmento que une um vértice ao ponto médio do lado
oposto a ele. A
3.3. Altura
AM é mediana relativa ao lado BC; Segmento que liga um dos vértices ao lado oposto (ou
P N BN é a mediana relativa ao lado AC; ao seu prolongamento) e que é perpendicular a esse
G
CP é a mediana relativa ao lado AB. lado (ou prolongamento).
Todo triângulo possui três alturas, que se encontram
B M C em um ponto chamado Ortocentro.
Baricentr
Todo triânguloo possui três medianas, que se 3.3.1. Posições do Ortocentro em relação a um
encontram em um ponto chamado baricentro. triângulo

3.3.1.1. é interno, se o triângulo é


acutângulo (todos os ângulos são agudos).
3.1.1. Base Média de um Triângulo
Def.: Base média de um triângulo é um 3.3.1.2. coincide com o vértice do ângulo
segmento que liga dois pontos médios dos lados de reto, se o triângulo é retângulo.
um triângulo. Este segmento é paralelo a um dos
lados e vale metade do lado do qual ele é paralelo. 3.3.1.3. é externo, se o triângulo é
A obtusângulo (possui um ângulo obtuso).
Sendo P, N e M pontos médios dos
lados AB, AC e BC, respectivamente,
temos:
P N  PN // BC  PN = Você seria capaz de encontrar o
G
 PM // AC  PM = Ortocentro de cada um dos triângulos
a seguir?
B M C  MN // AB  MN =

3.1.2. Propriedade do Baricentro


Sendo G o baricentro, temos:

 AG = AM ; GM = AM

 BG = BN ; GN = BN

 CG = CP ; PG = CP

11
PROF ANDRÉ FONSECA GEOMETRIA

h  altura do triângulo equilátero


3.4. Mediatriz
É uma reta perpendicular ao lado de um triângulo, Como O é também o baricentro (encontro das medianas) do
triângulo, esse ponto divide a altura AH em segmentos
passando pelo seu ponto médio. proporcionais a 2 e 1.
Todo triângulo possui três mediatrizes, que se Assim, se r e R são os raios das circunferências inscrita e
encontram em um ponto chamado Circuncentro. circunscrita, respectivamente, e h é a altura, é imediato que
A
No triângulo ao lado temos:
r  s é a mediatriz de BC; r = h e R = h
t  t é a mediatriz de AB;
P  r é a mediatriz de AC;
 P é o circuncentro do ABC.

B C

EXERCÍCIOS
3.4.1. Propriedade do Circuncentro
01. Observe a figura a seguir e determine:
3.4.1.1. O ponto P (circuncentro), no triângulo anterior,
é equidistante dos três vértices do triângulo. A
3.4.1.2. O circuncentro é também o centro da 1̂
circunferência circunscrita ao triângulo.
A

No triângulo ao lado temos que: 3̂


 m(AP) = m(CP) = m(BP); M
 AP, CP e BP são raios da N
P circunferência.

B C
a) os vértices do triângulo;
b) a indicação do triângulo;
c) o ângulo oposto ao lado PN;
Observações Importantes d) o lado oposto ao ângulo ;
e) ;
I. Num triângulo retângulo, a mediana relativa a
hipotenusa divide o triângulo original em dois f) a medida do ângulo se = 140º;
triângulos isósceles. g) a medida do ângulo se = 45º.

O ponto O é o centro da circunferência 02. É possível a construção de um triângulo retângulo


circunscrita ao triângulo. equilátero?
O
03. É possível a construção de um triângulo que tenha
dois ângulos retos? E a de um triângulo que tenha
dois ângulos obtusos?

04. No triângulo ABC a seguir, AM é a mediana.


II. No triângulo equilátero, o incentro, o baricentro, o Determine o perímetro desse triângulo.
ortocentro e o circuncentro coincidem num único
A
ponto O, chamado “centro do triângulo equilátero”.
A 3,5 cm
2,2 cm

R=
R 1,9 cm
h B M C
O
r=
r

B H C

Na figura acima temos:


R  raio da circunferência (maior) circunscrita ao triângulo
r  raio da circunferência (menor)inscrita ao triângulo
12
PROF ANDRÉ FONSECA GEOMETRIA

05. Considerando congruentes os triângulos abaixo,


calcule o valor de x e de y. 09. (PUC) Na figura abaixo, a = 100º e b = 110º.
Quanto mede o ângulo x?
a) 30º
5x - 2 68º b) 50º x
37 6x
37
c) 80º
54º d) 100º
68º a b
e) 220º
4y + 8 3y + 9

06. Na figura a seguir, 5x, 3x – 15º e 2x + 5º 10. (FAETEC) Na figura abaixo, r é a bissetriz do
representam as medidas dos três ângulos internos do ângulo ABC. Se  = 40º e  = 30º, então:
triângulo AMN. Nestas condições, qual deve ser o r a)  = 0º
valor de x? C
b)  = 5º
A c)  = 35º
 d)  = 15º
5x e) os dados são
  insuficientes para a
A B determinação de .
2x + 5º 3x – 15º
M
N
11. (UFMG) Na figura a seguir, , é
bissetriz de , é bissetriz de e a
07. Qual deve ser o valor da medida m do ângulo
externo do triângulo abaixo? medida do ângulo é 140º. A medida do ângulo
, em graus, é:
m
A

D
E
155º
110º
B C F
a) 35º d) 15º
08. (CAP-UFRJ-2006) Considere a figura a seguir. b) 40º e) 20º
c) 30º
C
12. Analise, em cada um dos casos abaixo, se é
possível construir um triângulo com as seguintes
45º
59º medidas dos lados:
B a) 6, 10 e 18
45º
b) 8, 4 e 6
86º c) 3, 10 e 17
62º
46º 59º 13. Classifique em verdadeira (V) ou falsa (F) cada
D afirmação a seguir:
O 60º 60º a) os ângulos agudos de um triângulo retângulo são
suplementares.
b) se a medida em graus de um ângulo é x, então o
60º seu suplemento mede, em graus, 180º - x.
A
c) qualquer triângulo isósceles tem todos os seus
ângulos agudos.
E d) se a medida em graus de um ângulo é x, então o
a) Calcule a medida do ângulo BÂO. seu complemento é 90º + x.
b) Identifique qual dos triângulos é um triangulo
retângulo. Justifique.
c) No triangulo CDO, identifique o lado de maior
comprimento. Justifique.
d) Determine o menor lado do polígono ABCDEO.

13
PROF ANDRÉ FONSECA GEOMETRIA

14. Na figura abaixo, a medida de AD é igual a 19. (UFF) O triângulo MNP é tal que M=80º e P=60º.
medida de BD. Então, x, y e z medem, A medida do ângulo formado pela bissetriz do ângulo
respectivamente: interno N com a bissetriz do ângulo externo P é:
B a) 20º b) 30º c) 40º d) 50º e) 60º

z y 20. (UFF) Determine o intervalo de variação de x,


para que exista um triângulo com as medidas:
a) x + 10, 2x + 4 e 20 – 2x.
b) 40 – x, 3x – 15 e x + 10
100º x 70º
A C (UFRJ-2000) Na figura ao lado, cada um
D 100º
dos sete quadros contém a medida de um
a) 100º, 30º e 40º ângulo expressa em graus. Em quaisquer
b) 100º, 70º e 10º três quadros consecutivos, temos os três x
c) 80º, 70º e 10º ângulos internos de um triângulo.
d) 80º, 30º e 40º Determine o valor de x. 65º

15. (PUC-RJ) Dada a figura, coloque os segmentos 21. Na figura, temos:


em ordem crescente.
d
30º a
50º
b 60º c e b
c
x
80º a) x = a – b + c
b) x = a + b + c
a c) x = a – b – c
d) x = – a + b + c
e) x = a + b – c

22. (UFU-MG) Na figura abaixo, AO e OB são


16. (UFMG) Na figura abaixo, DB = DE e AD é a perpendiculares, BC é bissetriz do ângulo DBA e AC é
bissetriz interna do triângulo ABC. O ângulo  mede: a bissetriz do ângulo EÂB. A medida do ângulo BCA é:
A
C
a) 10º
b) 14º D
c) 16º E
d) 18º 
e)20º 50º
46º B
B D C
O A E
17. (CP II) Na figura abaixo, os ângulos destacados
medem 30º, 45º e 60º. 23. (PUC-RJ) As dimensões do triângulo ABC são AB
Identifique, na figura, a medida de cada ângulo. = 11, AC = 18 e BC = 20. Calcule o perímetro do
triângulo AMN, sabendo-se que MN é paralelo a BC,
que OB é bissetriz do ângulo ABC e que OC é a
bissetriz do ângulo ACB. A

M O N
B C A

18. (UERJ/UFF) MNP é um triângulo isósceles (MN =


NP) cujo ângulo M vale 40º. I é o ponto de interseção B C
das bissetrizes internas do triângulo. O valor do
ângulo NIP é: 24. (UNB-DF) Considere as afirmações:
a) 35º b) 70º c) 90º d) 110º e) 140º I. Se num triângulo, a altura relativa a um lado
coincide com a bissetriz do ângulo oposto a ele, o
triângulo é necessariamente isósceles.

14
PROF ANDRÉ FONSECA GEOMETRIA

II. Num triângulo isósceles qualquer, as três 31. (PUC-RJ-2005) Os ângulos de um triângulo
medianas são necessariamente iguais. medidos em graus são:
III. Se um triângulo tem duas alturas iguais, então ele 3x – 48, 2x + 10 e x – 10.
é necessariamente equilátero. O maior ângulo mede:
Pode-se afirmar que: (A) 86° (B) 45° (C) 75° (D) 90° (E) 40°
a) I e II são corretas, III é falsa.
b) todas são falsas.
c) I é correta, II e III são falsas.
d) n.r.a.

25. Obtenha x na figura abaixo, onde as retas r e s


são paralelas.

x 2x + 20º
r

70º
s

26. Se a medida de um ângulo interno de um triângulo


é igual a soma das medidas dos dois outros ângulos
internos, então, necessariamente, este triângulo é:
a) retângulo
b) equilátero
c) tem lados medindo 3, 4 e 5;
d) é isósceles, não equilátero
e) tem ângulo interno de 30º

27. (FUVEST) As retas t e s são paralelas. A medida


do ângulo x, em graus, é:
t s

x
120º

140º

28. (PUC) O maior dos segmentos desenhados na


figura a seguir é:
B

59º
60º D

A 63º 57º

29. Em um triângulo AOE, os ângulos  e Ô medem,


respectivamente, 86º e 34º. Determine a medida do
ângulo formado pela mediatriz relativa ao lado OE e
pela bissetriz do ângulo Ê.

30. O triângulo AOE é tal que  = 80º e Ê = 60º. A 1-a) A, M, N; b) AMN; c)  ou MÂN; d) AN; e) 180º f) 40º; g) 135º; 2-Não.
Justifique.; 3-Não. Justifique.; 4-9,5cm; 5- x=10 e y=13; 6-x=19; 7-85º; 8-a)
medida do ângulo formado pela bissetriz interna de Ô 48º b) BOC c) CD; d) AB; 9-a; 10-b; 11-b; 12-a) não b)sim c) não; 13-
com a bissetriz externa do ângulo externo de Ê é: F,V,F,F; 14-d; 15-b<a<c<e<d; 16-c; 17-B=60º, C=45º, Â=30º; 18-d; 19-c; 20-
a)6/5<x<26/3, b)9<x<65/3; 21-x=15º; 22-m(BCA)=45º; 23-29; 24-c; 25-x=50;
a) 20º b) 30º c) 40º d) 50º e) 60º 26-a; 27-x=70º; 28-BD; 29-60º; 30-C;

15
PROF ANDRÉ FONSECA GEOMETRIA

QUADRILÁTEROS Propriedades:
Sendo o quadrado ao mesmo tempo losango e
retângulo, ele possui as propriedades desses outros
Quadrilátero é o polígono de quatro lados. dois polígonos.
A B
SOMA DOS ÂNGULOS INTERNOS 45º
A soma dos ângulos internos de qualquer quadrilátero 45º
convexo vale 360º.

PARALELOGRAMO M
Paralelogramo é o quadrilátero em que os pares de
lados opostos são paralelos.
A B D C
TRAPÉZIO
M Trapézio é o quadrilátero que possui apenas dois de
seus lados paralelos, denominados bases do trapézio.
D A distância entre as duas bases chama-se altura do
C
trapézio.

Trapézio Isósceles: os lados não paralelos são


iguais. Neste caso, os ângulos em cada base também
Propriedades: são iguais.
Em todo paralelogramo Trapézio Escaleno: se os lados não paralelos são
os lados opostos são sempre iguais; desiguais.
ângulos consecutivos são suplementares; e Trapézio Retângulo: se um dos lados não paralelos é
as diagonais cortam-se nos seus pontos médios. perpendicular às bases.

RETÂNGULO Propriedades:
Retângulo é o paralelogramo cujos ângulos são Num trapézio isósceles os ângulos adjacentes a
sempre iguais (portanto retos). uma mesma base são congruentes;
A B Num trapézio isósceles as diagonais são
congruentes.

Base Média (bm) de um trapézio


M
A base menor B
D C
Propriedades:
Em todo retângulo
as diagonais são congruentes; e M N
as diagonais se cortam nos pontos médios. bm

LOSANGO
Losango é o paralelogramo cujos lados são iguais.
D Base maior C
A B
A base média é paralela às bases e mede:

D
C
Propriedades:
Em todo losango Mediana de Euler
as diagonais são perpendiculares entre si; Chama-se Mediana de Euler de um trapézio ao
as diagonais se cortam nos pontos médios; e segmento que liga os pontos médios das diagonais e
as diagonais são bissetrizes dos ângulos iguais – que fica sobre a base média.
que neste caso, são os ângulos opostos. Seu comprimento mede:

QUADRADO
Quadrado é o paralelogramo que possui lados e
ângulos iguais.
16
PROF ANDRÉ FONSECA GEOMETRIA

A base menor B larguras respectivamente iguais a 60 cm e a


30 cm, conforme a figura:

30 Os degraus serão obtidos cortando-


se uma peça linear de madeira cujo
M N
P mE Q comprimento mínimo, em cm, deve
ser:
(A) 144. (B) 180. (C) 210.
(D) 225. (E) 240.
D Base maior C
60
39. (UFRJ-2003) De um retângulo de 18 cm de
EXERCÍCIOS largura e 48 cm de comprimento foram
retirados dois quadrados de lados iguais a 7
32. Calcule x em cada uma das figuras. cm, como mostra a figura.

a) b) 100º
x+50 2x – 30º
º x

x x – 20º 60º x
Qual o perímetro da figura resultante?
33. Calcule x no paralelogramo. 40. (UERJ-2001) O gráfico abaixo representa a
indicação da velocidade de um carro em
2x+10º
movimento, em função do tempo.
x +70º

34. Calcule x no paralelogramo.

150º

Sabendo-se que, em t = 2 s, a velocidade é de 6 m/s,


35. Na figura a seguir, A e B distam 2 cm e 6 cm, a ordenada do ponto A é:
respectivamente, da rata r, e M é o ponto (A) 3,5 (B) 3,0 (C) 2,5 (D) 2,0
médio de AB. Calcule a distância do ponto M
à reta r. 41. (ITA-SP) Dadas as afirmações:
I – Quaisquer dois ângulos opostos de um quadrilátero
B
são suplementares.
M II – Quaisquer dois ângulos consecutivos de um
paralelogramo são suplementares;
III – Se as diagonais de um paralelogramo são
A
r perpendiculares entre si e se cruzam em seu ponto
médio, então, este paralelogramo é um losango.
36.
(UNIFESP) Em um paralelogramo, as Podemos afirmar que:
medidas de dois ângulos internos a) todas são verdadeiras.
consecutivos estão na razão 1:3. b) apenas I e II são verdadeiras.
O ângulo menor desse paralelogramo mede: c) apenas II e III ao verdadeiras.
a) 45º b) 50º c) 55º d) 60º e) 65º d) apenas II é verdadeira.
e) apenas III é verdadeira.
37.
(UERJ-2000) Se um polígono tem todos os
lados iguais, então todos os seus ângulos GABARITO
internos são iguais. 21-a) 72º b) 80º; 22-60º; 23-75º; 24-4 cm; 25-a; 26-a;
Para mostrar que essa proposição é falsa, pode-se 27-D; 28-160 cm; 29-D; 30-C
usar como exemplo a figura denominada:
a) losango c) retângulo
b) trapézio d) quadrado

38. (ENEM-00) Um marceneiro deseja construir


uma escada trapezoidal com 5 degraus, de
forma que o mais baixo e o mais alto tenham
17
PROF ANDRÉ FONSECA GEOMETRIA

POLÍGONOS REGULARES
POLÍGONOS Nos polígonos regulares, podemos calcular o valor de
cada ângulo interno ou externo.
Chama-se polígono a toda linha poligonal simples.
Quando um polígono possui as medidas de seus lados ÂNGULOS INTERNOS
e ângulos congruentes ele é chamado de polígono
regular.

GÊNERO DE UM POLÍGONO
Chama-se gênero de um polígono o número de lados
que ele possui. ÂNGULOS EXTERNOS

número de lados (n) nome


n=3 triangulo
n=4 quadrilátero
n=5 pentágono
n=6 hexágono
OBS.: O número de diagonais que passam pelo centro
n=7 heptágono de um polígono regular é igual a n/2, quando n for um
n=8 octógono número par.
n=9 eneágono
n = 10 decágono EXERCÍCIOS
n = 11 undecágono
n = 12 dodecágono 42. Quantos lados tem o polígono no qual se pode
n = 15 pentadecágono traçar 35 diagonais de cada vértice?
n = 20 icoságono a) 35 b) 32 c) 10 d) 38 e) impossível

NÚMERO DE DIAGONAIS (d (d) 43. O número de diagonais que se pode traçar por
De qualquer vértice de um polígono de n lados partem um dos vértices de um icoságono é:
n – 3 diagonais. a) 10 b) 12 c) 17 d) 20 e) 170
O número total de diagonais é dado por:
44. Considere um polígono regular ABCD... . O
ângulo formado pelos prolongamentos dos
lados AB e DC mede 108º. Determine o
polígono.

ÂNGULOS EXTERNOS E INTERNOS 45. Em um polígono regular, o ângulo interno


Em qualquer vértice de um polígono a soma do ângulo mede 8x–20º e o externo mede 3x–20º. Qual
interno com o seu adjacente externo é sempre 180º. o polígono?
a) pentágono b) octógono c) decágono
d) hexágono e) eneágono
ae+ ai = 180º
ae 46. (ENEM-02) Na construção civil, é muito
ai comum a utilização de ladrilhos ou azulejos
com a forma de polígonos para o revestimento
de pisos ou paredes. Entretanto, não são
todas as combinações de polígonos que se
prestam a pavimentar uma superfície plana,
SOMA DOS ÂNGULOS INTERNOS (Si) sem que haja falhas ou superposições de
A soma de todos os ângulos internos de um polígono ladrilhos, como ilustram as figuras:
convexo é dada por

Si = 180º(n – 2)

SOMA DOS ÂNGULOS EXTERNOS (Se)


A soma de todos os ângulos externos de um polígono
convexo é
Se = 360º
Figura 1: Ladrilhos retangulares pavimentando o plano

18
PROF ANDRÉ FONSECA GEOMETRIA

e) acertou na premissa e errou nas conclusões.

50. (UFJF-MG) Em um pentágono convexo, os


ângulos internos formam uma P.A. de razão r.
O valor de r, tal que o maior ângulo desse
Figura 2: Heptágonos regulares não
pentágono meça 128º, é:
pavimentam o plano (há falhas ou
superposição) a) 10 b) 15º c) 20º d) 27º e) 36º

51. A figura descreve o movimento de um robô:

2 m 45º
A tabela traz uma relação de
alguns polígonos regulares, com as respectivas 2m
medidas de seus ângulos internos.
45º

Triângulo Quadrado Pentágono Hexágono 2m

partindo de A, ele, sistematicamente, avança 2m e


gira 45º para a esquerda.
Quando esse robô retornar ao ponto A, a trajetória
60º
percorrida terá sido:
90º
108º 120º a) uma circunferência.
b) um hexágono regular.
Se um arquiteto deseja c) um decágono regular.
Octógono Eneágono
utilizar uma combinação de d) um polígono não regular.
dois tipos diferentes de e) um octógono regular.
ladrilhos entre os polígonos
da tabela, sendo um deles GABARITO
octogonal, o outro tipo 31-D; 32-C; 33-decágono; 34-E; 35-B; 36-B; 37-36; 38-E; 39-A; 40-E
135º 140º escolhido deverá ter a
forma de um

A) triângulo B) quadrado
C) pentágono D) hexágono (E) eneágono

47.(CEFET) A soma de seis ângulos internos de


um octógono convexo é 880º. Se a diferença
entre os outros dois ângulos é de 20º, eles
valem, respectivamente,
a) 80º e 100º
b) 90º e 110º
c) 110º e 130º
d) 420º e 440º
e) 430º e 450º

48. (CAP-UFRJ) Se ABCDE é um pentágono


regular convexo, calcule a medida do ângulo
formado pelas diagonais AC e AD.

49. (CN) Um aluno declarou o seguinte, a respeito


de um polígono convexo P de n lados:
“Partindo da premissa de que eu posso traçar (n – 3)
diagonais de cada vértice de P, então, em primeiro
lugar, o total de diagonais de P é dado por n(n – 3); e,
em segundo lugar, a soma dos ângulos internos de P
é dada por 180º(n – 3)”.
Logo, o aluno:
a) errou na premissa e nas conclusões;
b) acertou na premissa e na primeira conclusão, mas
errou na segunda conclusão;
c) acertou na premissa e na segunda conclusão, mas
errou na primeira conclusão;
d) acertou na premissa e nas conclusões
19
PROF ANDRÉ FONSECA GEOMETRIA

Quando, de um ponto exterior, traçamos duas retas


CIRCUNFERÊNCIA tangentes a uma circunferência, os segmentos
compreendidos entre o tal ponto e os pontos de
tangencia são congruentes.
Circunferência de círculo, ou simplesmente
circunferência, é o lugar geométrico dos pontos do A PA = PB
plano que equidistam de um ponto fixo.
O ponto fixo é chamado centro e a distância comum é P
o raio da circunferência.

B
raio O ângulo formado na circunferência pelo raio e a
O
reta tangente mede 90º.

O
PRINCIPAIS ELEMENTOS DA CIRCUNFERÊNCIA
E B

A F TEOREMA DE PITOT
Se um quadrilátero é circunscritível a uma
circunferência, as somas das medidas dos lados
C D opostos são iguais. A
O
r B
s
t
AB + CD = BC + DA

D C
Na figura acima, temos.
ÂNGULO CENTRAL E MEDIDA ANGULAR DE UM ARCO
O é o centro; O ângulo formado no centro da circunferência por dois
AB é uma corda; raios chama-se ângulo central, e sua medida é a
AB é um arco; mesma (medida angular) do arco formado pelos raios.
FE é uma flexa
A
CD é o diâmetro;
r é a reta secante à circunferência; 
s é a reta tangente à circunferência; O
t é a reta exterior à circunferência. B

OBS.: Círculo é a superfície plana limitada pela


circunferência. ÂNGULO INSCRITO EM UM ARCO
A
PROPRIEDADES IMPORTANTES 
A mediatriz de uma corda sempre passa pelo 
centro da circunferência, e pelo ponto médio do arco
subtendido pela corda. 
E B B
A F Os ângulos, ,  e  estão inscritos no arco AB da
figura.
O
Consequências Importantes
Se um triangulo está inscrito em uma
circunferência, de tal forma que um de seus lados seja
Duas cordas paralelas de uma circunferência um diâmetro, então, este lado é a hipotenusa do
sempre determinam, entre suas extremidades, arcos triangulo, que é retângulo.
B A
congruentes.
D
A
C r r B
C
O O

20
PROF ANDRÉ FONSECA GEOMETRIA

Para que um quadrilátero seja inscritível em uma


circunferência, deve ter ângulos opostos 53. (UFRJ) Na figura abaixo, o pentágono ABCDE
suplementares. A
é circunscrito ao círculo de centro O e F, G, H,
B I e J são pontos de tangência.
Os segmentos AF, CH e DI têm o mesmo
comprimento x. EJ mede x + 1, enquanto BG mede x
+ 2. O perímetro do pentágono é igual a 16 e a
D distância OB é igual a 7. A
C F G
ÂNGULO DE SEGMENTO E
É o ângulo formado por uma corda e uma reta
tangente à uma circunferência em um dos extremos B
O
da corda. J
A
D I H
C
Determine o raio do circulo inscrito no pentágono.

54. Determine o perímetro do triangulo da figura
B
abaixo.
ÂNGULO EXCÊNTRICO INTERNO
A D

B
C
2 6
55. cm cm
(PUC-MG) na figura abaixo, o valor de x é:
ÂNGULOS EXCÊNTRICOS EXTERNOS
D
m C
3x+12º
m x º
n  n
 E B

a) 15º b) 18º c) 24º d) 32º e) 35º

56. (CESGRANRIO-RJ) Se, na figura, m(AB) =


20º, m(BC) = 124º, m(CD) = 36º e m(DE) =
m n 90º, então o ângulo xEmede:

D A
x
B
C
EXERCÍCIOS
a) 34º b) 35º30’ c) 37º d) 38º30’ e) 40º
52. Na figura, R, S e T são pontos de tangencia do
circulo inscrito ao triangulo ABC. SE AB = 6 57. Calcule o raio da circunferência inscrita em
cm, AC = 7 cm e BC = 9 cm, determine AR, BS e um triangulo retângulo de catetos 3 e 4.
CT.
A 58. Considere o círculo que tangencia a reta r no
T ponto A e tem seu diâmetro BC sobre a reta s.
R
Então, os ângulos do triangulo ABC valem:

B S C
B C
20º O

a) 35º, 55º e 90º d) 40º, 60º e 80º


b) 20º, 70º e 90º e) 50º, 50º e 80º
c) 30º, 60º e 90º
21
PROF ANDRÉ FONSECA GEOMETRIA

59. Na figura abaixo, O é o centro do círculo e CD


= BO. Calcule o ângulo .
B
C

72º 
A D
O E

a) 18º b) 36º c) 24º d) 9º e) 16º

60.A e B são pontos de um círculo que o dividem


em arcos proporcionais a 5 e 4. As tangentes
traçadas por A e B formam:
a) 60º b) 50º c) 40º d) 30º e) 20º

61. Calcule os ângulos ,  e  da figura abaixo,


sabendo-se que t é tangente ao círculo e que
 = 6.



O 

t
a)  = 10º,  = 120º e  = 60º
b)  = 15º,  = 160º e  = 90º
c)  = 20º,  = 120º e  = 120º
d)  = 15º,  = 150º e  = 90º
e)  = 15º,  = 165º e  = 90º

GABARITO
41-AR=1cm, BS=5cm e CT=4cm; 42-210; 43-24cm; 44-C; 45-C;
46-1 cm; 47-B; 48-C; 49-E; 50-D

22
PROF ANDRÉ FONSECA GEOMETRIA

Imagine que uma foto 3x4 (3 cm de largura


LINHAS PROPORCIONAIS por 4 cm de comprimento) deve ser ampliada de modo
que o lado maior meça 20 cm. Dessa forma, o lado
Antes de começar a discussão sobre o menor deverá medir 15 cm.
assunto de que trataremos aqui, vamos compreender Veja.
o significado de algumas expressões que serão muito
utilizadas no desenvolver dessa questão.

ALGUMAS DEFINIÇÕES... Se simplificarmos a segunda fração, veremos


que a fração resultante será a mesma que a primeira
Razão
fração, portanto, são equivalentes. Neste caso, temos
Razão é o quociente entre dois valores, ou uma proporção acontecendo.
seja, a divisão ou comparação entre eles.
Podemos ter como exemplo de razão a escala
de um mapa. Veja:
1 : 100.000 cm
TEOREMA DE THALES
Na escala acima, vemos uma comparação
entre o tamanho representado pelo mapa Um feixe de retas paralelas ( a, b, c e d na figura)
(representação de um espaço através de um figura) e determina em duas transversais quaisquer, (r e s na
o tamanho real. O “tamanho” a que se refere aqui figura) segmentos proporcionais.
pode ser, por exemplo, a distância entre duas cidades.
A escala (que é uma razão) acima nos diz que se a r s
distância entre duas cidades no mapa é de 1 cm,
então, a distância real entre as duas cidades é de A A’
a Na figura, a//b//c//d.
100.000 cm, ou seja, 1km.
A razão que nos interessa aqui é a razão entre
dois segmentos. Razão entre dois segmentos é o B B’ b
quociente entre os números que exprimem suas
medidas, sendo sempre o segundo número diferente C C’
c
de zero, ou seja, dado um número a e um número b, a
razão entre a e b é representada por
D D’
. (lê-se: a está para b ou simplesmente a d

; b≠0
Podemos escrever, para a figura acima:
para b).

A razão entre os lados de um televisor padrão


widescreen é 16:9, ou seja, a cada 16 cm de
comprimento, temos 9 cm de largura.
Vale ressaltar que a mesma razão acima pode ser CONSEQUÊNCIAS DO TEOREMA DE THALES
também escrita na forma .
1ª consequência:
Quando uma reta paralela a um lado de um
triangulo intercepta os outros lados em dois pontos
Proporção distintos, ela determina sobre esses lados segmentos
Proporção é a igualdade (equivalência) entre proporcionais.
duas razões. Dois segmentos são proporcionais a A
outros dois quando a razão entre os dois primeiros é r//BC  EF//BC
igual à razão entre os outros dois, ou seja, sejam a, b, 
c e d números inteiros quaisquer; dizemos que esses
números formam, nesta ordem, uma proporção, se E F r
.
B C
Exemplo numérico: .

Uma situação onde podemos verificar uma proporção


acontecendo é quando queremos ampliar uma
fotografia, seja digitalmente ou analogicamente.

23
PROF ANDRÉ FONSECA GEOMETRIA

2ª consequência: Você deve ter percebido que não, não pela


A bissetriz de um ângulo interno de um triângulo dificuldade visual, mas pelo conceito do “igual”.
divide o lado oposto em segmentos proporcionais aos Quando dizemos que duas figuras são iguais, elas
lados adjacentes. devem ter todas as características em comum,
A inclusive o tamanho. E isso, é fácil verificar que não
acontece com as duas figuras anteriores.
â Contudo, mesmo não podendo dizer que são iguais,
â podemos dizer que são figuras semelhantes.

Duas figuras são semelhantes quando possuem a


C
mesma forma, ou seja, quando possuem mesmos
B D ângulos e os lados correspondentes dessas figuras
são proporcionais.
Observações:
Se dois triângulos são semelhantes e a razão de Quando ampliamos ou reduzimos uma figura de modo
semelhança é k, então: que essa mudança aconteça de forma proporcional,
obtemos assim duas figuras semelhantes.
 a razão entre duas alturas correspondentes é k. Voltando às figuras anteriores...
 a razão entre duas medianas correspondentes é k. Você notou que o comprimento da figura maior é o
 a razão entre duas bissetrizes correspondentes é k. produto (multiplicação) entre o comprimento da
 a razão entre seus perímetros é k. primeira figura e um número k?
 a razão entre suas áreas k2. Você notou que o mesmo acontece com a largura?
Esta é uma característica presente em figuras que
são semelhantes – seja uma ampliação ou redução de
uma outra figura, isso só acontece multiplicando ou
Semelhança aplicada a Triângulos dividindo ambos os lados por um mesmo número – e k
é chamado de razão de semelhança entre as figuras
Dizemos que duas figuras – ou dois objetos – são ou constante de proporcionalidade.
semelhantes quando se parecem. Uma lapiseira e
uma caneta, por exemplo, são parecidos e, por isso,
poderíamos dizer “são semelhantes”. SEMELHANÇA DE TRIÂNGULOS
Mas, o conceito que semelhança que será
abordado aqui não é o mesmo do dia-a-dia, o mesmo Dois triângulos são semelhantes quando tiverem:
que poderíamos aplicar à lapiseira e à caneta, mas  ângulos correspondentes congruentes;
trataremos aqui o conceito de semelhança do ponto  lados correspondentes proporcionais.
de vista da Geometria, ou seja, da Matemática.
Observe as imagens a seguir. Se um triângulo ABC é semelhante a outro DEF,
indicamos que são semelhantes da seguinte forma:
ABC ~ DEF

Observe os triângulos representados pelas figuras a


seguir. D

A 35º

b b k 35º

65º 80º
B C E F

a
Pelas representações, verificamos que ABC ~
DEF, já que E
a k temos que .

O que você diria sobre as figuras acima?


São iguais?
Se você respondeu que sim, experimente pegar uma Observe os exercícios resolvidos a seguir.
régua e medir os lados de cada uma delas. E então,
são iguais.

24
PROF ANDRÉ FONSECA GEOMETRIA

Exercício 1
As retas r1, r2 e r3, na figura a seguir, são paralelas 64. Determine o valor de x e y nas figuras.
e as medidas dos segmentos de transversais são
dadas em centímetros. Nessas condições, calcule o a) b)
valor de x. t1 t2 A A

x 6
4 y
r1 N
x 1,2 M
10 y
r2 M N 4
5 B
4 6 C
15 3 15
r3 B x C

c) d)
Solução: 21 D
y E
12 A
 3 x = 1,2  15  3x = 18  x =  D
A 8 20
12 B
y x C
x = 3 cm. x 6

B 5 E 4 C

Exercício 2
Qual o valor de x na figura a seguir?
D

A 35º
65. Determine a medida do lado do quadrado da
35º x 39 cm figura abaixo.
12 cm 13 cm

65º 80º
C E F 4
B 30 cm
10 cm

66. (Enem) A sombra de uma pessoa que mede 1,80


XERCÍCIOS m de altura mede 60 cm. No mesmo momento, a seu
lado, a sombra projetada de um poste mede 2,00 m.
62. Para medir a altura de um pinheiro, fiz o Se, mais tarde, a sombra do poste diminui 50 cm, a
seguinte: peguei um bastão de 1,5 m e verifiquei sombra da pessoa passou a medir:
que ele projetava uma sombra de 2 m, enquanto o a) 30 cm b) 45 cm c) 50 cm d) 80 cm e) 90 cm
pinheiro projetava uma sombra de 16m. Que
altura encontrei?
67. Observe as afirmações a seguir.
I. Todos os triângulos congruentes são semelhantes.
h II. Todos os triângulos semelhantes são congruentes.
III. Todos os triângulos retângulos são semelhantes.
1,5 m IV. Todos os triângulos equiláteros são semelhantes.
V. Dois triângulos isósceles que têm os ângulos do
vértice congruentes são semelhantes.
2m A respeito das afirmações, podemos dizer que:
16 m a) todas são verdadeiras.
b) todas são falsas.
c) uma delas é verdadeira.
63. (CAP-UFRJ-06) Na figura d) duas delas são verdadeiras.
ao lado, o ABC é retângulo e e) três delas são verdadeiras.
isósceles.
Considere AP = 3 e PQ = 6.
a) Determine a medida do 68. (UNIRIO) Numa cidade do interior, à noite, surgiu
segmento AQ. um objeto voador não identificado, em forma de disco,
b) Determine o perímetro do que estacionou a 50 m do solo, aproximadamente. Um
ABC. helicóptero do exercito, situado a aproximadamente 30
25
PROF ANDRÉ FONSECA GEOMETRIA

m acima do objeto, iluminou-o com um holofote, sucessivamente, por R, Q, P, T, retornando,


conforme mostra a figura. finalmente, a S. Assinale a opção que indica o
perímetro do circuito.
Sendo assim, pode-se a) 4,5km b) 19,5km c) 20,0km d) 22,5km e) 24,0km
afirmar que o raio do disco-
voador mede, em m,
30 m aproximadamente: 72. Na figura, AB e DE são paralelos. Calcule o valor
a. 3,0 de x.
b. 3,5 A B
c. 4,0
50 m d. 4,5 7 cm 6 cm
e. 5,0 C
sombra
5 cm x
16 m

D E
69. (UFRJ – Específica) Três goiabas perfeitamente
esféricas de centros C1, C2 e C3 e raios 2 cm, 8 cm e 2
cm estão sobre uma mesa tangenciando-se como 73. A razão de semelhança de dois triângulos
sugere a figura abaixo. equiláteros é 2/5. O lado do menor mede 8 m. Calcule
Um bichinho que a medida do lado do outro triângulo.
está no centro da
primeira goiaba 74. Na figura, AD é a bissetriz interna do ângulo A.
C2 quer se dirigir Calcule o valor de x.
para o centro da
C1 terceira pelo A
C3
caminho mais
curto.
8 cm
4 cm
Quantos centímetros percorrerá?

70. (UEL) O gráfico a seguir mostra a atividade de B 2 cm D x C


café, em milhões de toneladas, em certo município do
estado do Paraná.
14
75. Um triângulo, cujos lados medem 12 m, 18 m e
20m, é semelhante a outro cujo perímetro mede 10 m.
Calcule as medidas dos lados do triângulo menor.
5

76. Um menino de 1,50 m de altura observa, num dia


de sol, as sombras de uma torre de radio-emissora e a
anos
1990 1996 sua própria sombra. Não dispondo de uma fita métrica
ou de trena, ele toma um cordão, mede sua sombra e
De acordo com o gráfico, é correto afirmar que, em a compara com a da torre, verificando ser esta 10
1994, a produção de café nesse município foi, em vezes a medida da sua. Calcule a altura da torre.
milhões de toneladas:
a) 9,5 b) 9 c) 10,5 d) 11 e) 12,5
77. Um menino de 1,50 m de altura observa, num dia
de sol, as sombras de uma torre de radio-emissora e a
71. (UFF-2002) O circuito triangular de uma corrida sua própria sombra. Não dispondo de uma fita métrica
está esquematizado na figura a seguir. ou de trena, ele toma um cordão, mede sua sombra e
a compara com a da torre, verificando ser esta 10
P Rua PQ Q Av. QR R
4 km
vezes a medida da sua. Calcule a altura da torre.
2 km
3 km
Av. SR

S
Rua TS
GABARITO:
T
62-12m; 63-18+92; 64-a) x=10 e y=6 b) x=8; y=3 c) x = 10/3 e y =
As ruas TP e SQ são paralelas. Partindo de S, cada 20/3 d) x = 18; y = 14; 65-2,4; 66-B; 67-E; 68-A; 69-16,8 cm; 70-D;
71-B; 72-35/6; 73-20m; 74-4 cm; 75-12/5, 18/5 e 4 cm; 76-15m; 77-
corredor deve percorrer o circuito passando, 15m
26
PROF ANDRÉ FONSECA GEOMETRIA

Exemplo de aplicação do Teorema de Pitágoras:

RELAÇÕES MÉTRICAS NO (FAETEC-05) No topo de uma caixa d’água distante 6m do


solo, apóia-se uma escada, como na figura abaixo.
TRIÂNGULO RETÂNGULO
TRIÂNGULOS RETÂNGULOS A
6m

Â1
b Â2 c
h

8m
C H B
m n Se a distância da viga de sustentação da caixa d’água até o
a pé da escada é de 8 m e a sua altura é de 6 m, o
comprimento dessa escada, em metros é:
Solução:
O ABC acima é chamado “triângulo retângulo” Chamando o comprimento da escada de x, temos:
porque possui um ângulo interno reto, ou seja, 90º. a2 = b2 + c2
Os lados perpendiculares entre si – que formam o x2 = 6 2 + 8 2
ângulo reto – denominam-se catetos. O lado oposto x2 = 36 + 64
ao ângulo reto (maior ângulo) é chamado hipotenusa. x2 = 100
x = 100
No triângulo retângulo acima, no qual  é o ângulo
reto, temos: x = 10
Resposta: 10 metros
a é a medida da hipotenusa BC.
c é a medida do cateto AB. EXERCÍCIOS
b é a medida do cateto AC.
h é a medida da altura AH. 78. Em um triângulo retângulo os catetos medem 15cm
m é a projeção do cateto b sobre a hipotenusa. e 20cm. Ache as medidas de sua hipotenusa.
n é a projeção do cateto c sobre a hipotenusa.
79. Em um triângulo retângulo os catetos medem 15cm
Em relação aos ângulos, temos as relações: e 20cm. Ache as medidas de sua hipotenusa.

ou 80. (UFPE) Um barco navegou 10 km para o oeste,


depois 5 km para o sul, depois 13 km para o leste, e
Em relação aos lados, temos as relações: finalmente 9 km para o norte.
Onde o barco parou relativamente ao ponto de
1) a = m + n partida?
2) b2 = am a) 5 km ao norte
3) c2 = na b) 3 km a sudeste
c) 4 km ao sul
4) h2 = mn
d) 3 km a sudoeste
5) ah = bc e) 5 km a nordeste
TEOREMA DE PITÁGORAS 81. Millôr Fernandes, em uma bela homenagem à
Em qualquer triângulo retângulo vale a relação Matemática, escreveu um poema do qual extraímos o
fragmento abaixo:

a2 = b2 + c2 Às folhas tantas de um livro de Matemática,


a um Quociente apaixonou-se um dia doidamente
b por uma Incógnita.
Olhou-a com seu olhar inumerável
e viu-a do ápice à base: uma figura ímpar;
c olhos rombóides, boca trapezóide,
corpo retangular, seios esferóides.
Fez da sua uma vida paralela à dela,
até que se encontraram no Infinito.
"Quem és tu?" – indagou ele em ânsia radical.
"Sou a soma dos quadrados dos catetos.
Mas pode me chamar de hipotenusa."
...................................................................................
(Millôr Fernandes. Trinta Anos de Mim Mesmo.)

27
PROF ANDRÉ FONSECA GEOMETRIA

A Incógnita se enganou ao dizer quem era. Para 86. Use o teorema de Pitágoras para encontrar a
atender ao Teorema de Pitágoras, deveria dar a fórmula da diagonal do cubo abaixo.
seguinte resposta:
a) "Sou a soma dos catetos. Mas pode me chamar de
hipotenusa." d a
b) "Sou o quadrado da soma dos catetos. Mas pode
me chamar de hipotenusa."
c) "Sou o quadrado da soma dos catetos. Mas pode a
me chamar de quadrado da hipotenusa." a
d) "Sou a soma dos quadrados dos catetos. Mas pode
me chamar de quadrado da hipotenusa." 87. As circunferências da figura abaixo possuem raios
R e r. Calcule o segmento da tangente comum.
82. A medida da altura do trapézio a seguir é:

12

10 h 10

24
x
a) 9,5 b) 9 c) 8,5 d) 8

83. (UERJ-2000) Observe o desenho. 88. Considere a figura a seguir na qual os segmentos
Ele representa uma folha de reta AB e CD são perpendiculares ao segmento de
retangular com 8cm x reta BC. Se AB = 19 cm, BC = 12 cm e CD = 14 cm,
13cm, que foi recortada determine a medida, em cm, do segmento de reta AD.
formando duas figuras I e
II, que, apesar de
A
distintas, possuem a
mesma área.
A diferença entre o
perímetro da figura I e da D
figura II, em cm,
corresponde a:
a) 0 b) 2 c) 4 d) 6

84. (UFF) Na figura abaixo, os triângulos ABC e DEF


são equiláteros. B C

B
89. Para trocar uma lâmpada, Roberto encostou uma
E escada na parede de sua casa, de forma que o topo
da escada ficou a uma altura de 14 metros (figura 1).
Enquanto Roberto subia os degraus, a base da
escada escorregou por 1 metro, indo tocar o muro
A C D F paralelo à parede (figura 2). Refeito o susto, Roberto
reparou que, após deslizar, a escada passou a fazer
Sabendo que AB, CD e DE medem, respectivamente, um ângulo de 45º com a horizontal (figura 2).
6m, 4m e 4m, calcule a medida de BE. Pergunta-se:

85. (UFRJ-99-2ª fase) Na figura, o triângulo AEC é


equilátero e ABCD é um quadrado de lado 2 cm.

Calcule a distância
parede

parede

BE.
muro

muro

45º

Figura 1 Figura 2

28
PROF ANDRÉ FONSECA GEOMETRIA

A) qual é a distância entre a parede da casa e o


muro?
B) Qual é o comprimento da escada de Roberto?

90. Os semicírculos de diâmetro AO, OB e AB têm


centros sobre a reta AB. O círculo de centro O lhes é
tangente. Se AB = 12 cm, calcule R.

R

A O B

a) 1 cm
b) 3 cm
c) 5 cm
d) 2 cm
e) 4 cm

91. (ENEM-2006) Observe a figura.

Na figura acima, que representa o projeto de uma


escada com 5 degraus de mesma altura, o
comprimento total do corrimão é igual a
A) 1,8 m.
B) 1,9 m.
C) 2,0 m.
D) 2,1 m.
E) 2,2 m.

92. (ENEM-2005) Quatro estações distribuidoras de


energia A, B, C e D estão dispostas como vértices de
um quadrado de 40 km de lado. Deseja-se construir
uma estação central que seja ao mesmo tempo
equidistante das estações A e B e da estrada (reta)
que liga as estações C e D.
A nova estação deve ser localizada
(A) no centro do quadrado.
(B) na perpendicular à estrada que liga C e D
passando por seu ponto médio, a 15 km dessa
estrada.
(C) na perpendicular à estrada que liga C e D
passando por seu ponto médio, a 25 km dessa
estrada.
(D) no vértice de um triângulo equilátero de base AB,
oposto a essa base.
(E) no ponto médio da estrada que liga as estações A GABARITO
e B. 61-25 cm; 62- – 2 + 22 ou – 2(1 – 2); 63-E; 64-D; 65-D; 66-D; 67-
BE = 84 ou 221; 68-6 – 2; 69-a3; 70-2Rr; 71-13cm; 72- a) 3m
b) 32 m; 73-D; 14-D; 15-B
29
PROF ANDRÉ FONSECA GEOMETRIA

ÁREA DAS PRINCIPAIS FIGURAS PLANAS


área de Figuras Planas
Área do Paralelogramo

h Slosango = b  h
O CONCEITO DE ÁREA
Em diversos contextos e por vários motivos os povos
sentiram necessidade de medir superfícies. No antigo Egito, por
b
exemplo, a cada ano, os estiradores de cordas – homens
incumbidos de demarcar as terras inundadas pelo rio Nilo,
determinavam a área de cada propriedade, não apenas para que Áreas de Triângulos
os proprietários pudessem preservar suas terras, mas também e
principalmente, para que fosse garantido o pagamento dos
impostos sobre essas propriedades aos faraós. Conhecendo sua base e sua altura
Atualmente, por outros motivos, temos a necessidade de
determinar áreas. Por exemplo, ao fazer a previsão de gastos para
azulejar uma cozinha, ou, ainda, para decidir qual a área que um
escritório deve ter para acomodar uma determinada quantidade de STriângulo =
funcionários. h
É importante saber que para medir* uma superfície é
necessário tomar uma outra superfície, considerada unidade de
medida, e verificar quantas vezes essa superfície cabe naquela
b
que se quer medir.
* um dos significados para a palavra medir é comparar. Ao medir
uma superfície, estamos, na verdade, comparando-a com outra, que Conhecendo o lado (l) de um triângulo equilátero
pode ser menor ou maior.

NOTAS
STriângulo =
Unidade de medida é o instrumento usado para comparar
medidas. Por exemplo, o m2, a polegada, a jarda o ano-luz etc.
Perímetro de um polígono é a soma das medidas dos seus
lados.
Figuras congruentes têm mesma área. l
Se uma região R é dividida em duas regiões R 1 e R2, então,
SR = SR1 + SR2.
A área de uma região retangular é o produto dos
Conhecendo seus lados – Fórmula de Hierão
comprimentos de dois lados consecutivos.
c
Sretângulo = b  h h
a
b
b
Duas figuras são ditas equivalentes quando têm áreas iguais.
Um metro quadrado (1m2) é uma superfície que corresponde STriângulo =
a um quadrado de 1 metro de lado.
p é o semi-perímetro do triângulo

Conhecendo dois lados e o ângulo entre eles

b STriângulo =


c

30
PROF ANDRÉ FONSECA GEOMETRIA

Área do Losango EXERCÍCIOS


A B
93. (UFRJ - 2004) Um grande ato publico em favor da
Educação foi organizado em uma certa cidade. Uma
SLosango = avenida de 1,25 km de extensão e 40 m de largura foi
D totalmente tomada pelo público.
C
Supondo que quatro pessoas ocupam 1 metro
quadrado, calcule quantas pessoas foram ao evento.

Área do Trapézio 94. (Unirio-2002) Considere um tablado para a Escola


de Teatro da UNIRIO com a forma trapezoidal abaixo.
A base menor B

STrapézio = 15 m

5m 5m
M N
bm
9m

C Quantos m2 de madeira serão necessários para cobrir


D Base maior a área delimitada por esse trapézio?
a) 75 b) 36 c) 96 d) 48 e) 60

Área do Círculo de raio R


95. Calcule a área sombreada, sabendo que a área do
paralelogramo é 80 m2.
SCírculo =

raio
O A

96. (UENF 2000) Um terreno com a forma de um


Área do Setor Circular de raio R e ângulo  quadrado de 40m de lado foi dividido em três regiões
retangulares, destinadas à construção de uma casa
(I), um campo de futebol (II) e uma piscina (III),
conforme sugere a figura a seguir:

(I)
40 m (II)

O (III) x
R
25 m

Sabendo que as áreas das regiões I e II são iguais,


calcule:
A) a área da região II;
B) o valor de x na região III.

97. (Enem) Um terreno com o formato mostrado na


figura foi herdado por quatro irmãos e deverá ser
dividido em quatro lotes de mesma área. Um dos
irmãos fez algumas propostas de divisão para que
fossem analisadas pelos demais herdeiros.
Dos esquemas abaixo, onde lados de mesma medida
têm símbolos iguais, o único em que os quatro lotes
não possuem, necessariamente, a mesma área é:

31
PROF ANDRÉ FONSECA GEOMETRIA

101. (ENEM-03) Dados divulgados pelo Instituto


Nacional de Pesquisas Espaciais mostraram o
processo de devastação sofrido pela Região
Amazônica entre agosto de 1999 e agosto de 2000.
Analisando fotos de satélites, os especialistas
concluíram que, nesse período, sumiu do mapa um
total de 20000 quilômetros quadrados de floresta. Um
órgão de imprensa noticiou o fato com o seguinte
texto:
O assustador ritmo de destruição é de um
campo de futebol a cada oito segundos.
Considerando que um ano tem aproximadamente
32x106 s (trinta e dois milhões de segundos) e que a
medida da área oficial de um campo de futebol é
aproximadamente 10-2 km2 (um centésimo de
quilômetro quadrado), as informações apresentadas
nessa notícia permitem concluir que tal ritmo de
desmatamento, em um ano, implica a destruição de
uma área de
A) 10 000 km2, e a comparação dá a ideia de que a
devastação não é tão grave quanto o dado numérico
nos indica.
98. (UFRJ) Na figura abaixo o quadrado ABCD tem B) 10 000 km2, e a comparação dá a ideia de que a
lado 6. Q1, Q2, Q3 e Q4 são quadrados de lado x. A devastação é mais grave do que o dado numérico nos
região hachurada tem área 16. indica.
C) 20 000 km2, e a comparação retrata exatamente o
ritmo da destruição.
D) 40 000 km2, e o autor da notícia exagerou na
comparação, dando a falsa impressão de gravidade a
um fenômeno natural.
(E) 40 000 km2 e, ao chamar a atenção para um fato
realmente grave, o autor da notícia exagerou na
comparação.

Determine x. 102. (ENEM-01) Um município de 628 km² é atendido


por duas emissoras de rádio cujas antenas A e B
99. (PUC-SP) Uma pizzaria oferece aos seus clientes alcançam um raio de 10km do município, conforme
pizzas grandes de forma circular por R$ 50,00. Para mostra a figura:
atender alguns pedidos, a pizzaria passará a oferecer Para orçar um contrato
a seus clientes pizzas médias, também de forma publicitário, uma
circular. Qual deverá ser o preço da pizza média, se o agência precisa avaliar
preço das pizzas são proporcionais às suas áreas? a probabilidade que um
(Dados: raio da pizza grande = 35 cm e raio da pizza morador tem de,
média = 28 cm). circulando livremente
pelo município,
encontrar-se na área
de alcance de pelo menos uma das emissoras.
100. (UFF-2000) Os retângulos R1, R2 e R3, Essa probabilidade é de, aproximadamente,
representados na figura, são congruentes e estão A) 20% B) 25% C) 30% D) 35% E) 40%
divididos em regiões de mesma área.
103. (UERJ-2003) Uma folha de papel retangular,
como a da figura 1, de dimensões 8 cm × 14 cm, é
dobrada como indicado na figura 2.
R1 R2 R3,
Ao se calcular o quociente entre a área da região
pintada e a área total de cada um dos retângulos R1,
R2 e R3, verifica-se que os valores obtidos formam
uma progressão geométrica (P.G.) decrescente de
três termos.
A razão dessa P. G. é:
a)1/8 b) 1/4 c) 1/2 d) 2 e) 4

32
PROF ANDRÉ FONSECA GEOMETRIA

Em relação ao prisma, considere:


Se o comprimento CE é 8 cm, a área do polígono - cada um dos ângulos A, B, C e D da base superior
ADCEB, em cm2,é igual a: mede 120º;
a) 112 b) 88 - as arestas AB, BC e CD medem 10 cm cada.
c) 64 d) 24 Considere, ainda, que o papelão do qual é feita a
embalagem custa R$10,00 por m2 e que 3 = 1,73.
104. (Unesp-SP) O menor país do mundo é o Vaticano, Na confecção de uma dessas embalagens, o valor,
com 0,4 km2 de área. Se o território do Vaticano em reais, gasto somente com o papelão é
tivesse a forma de um quadrado, então a medida de aproximadamente igual a:
seus lados, em metros, estaria entre: a) 0,50 b) 0,95 c) 1,50 d) 1,85
a) 200 e 201 b) 220 e 221
c) 401 e 402 d) 632 e 633
108. (UFF-04) Dentre as previsões populacionais para
105. (UFRJ - 2005) Uma pizzaria vende pizzas grandes o Brasil, a mais sensata parece ser a do Fundo das
e pequenas no tradicional formato circular. As grandes Nações Unidas. Essa instituição prevê que o país
têm 40 cm de diâmetro e custam R$ 18,00; as estacionará em torno de 400 milhões de habitantes,
pequenas têm 20 cm de diâmetro e custam R$ 6,00. no fim do século XXI.
Todas têm a mesma espessura. Trecho adaptado de reportagem da revista Veja, 27 de março de 1996.
a) Lúcia e Raquel foram a essa pizzaria dispondo,
A mesma reportagem considera, ainda, que tal
cada uma, de R$ 10,00. Raquel propôs dividir uma
crescimento populacional garantiria ao Brasil uma
pizza
densidade demográfica (razão entre o número de
grande; Lúcia sugeriu que pedissem três pequenas.
habitantes e a área do país), no fim do século XXI,
Qual dessas opções permite que elas comam mais?
igual à metade da densidade demográfica da França
b) Manuel e Joaquim foram a essa pizzaria, com muita
no ano de 1996.
fome, e gastaram R$ 60,00 em 10 pizzas pequenas.
Sabe-se que a área territorial do Brasil é,
Determine de quantas outras formas eles poderiam,
aproximadamente, 15,5 vezes a área da França.
nessa pizzaria, gastar os mesmos R$ 60,00 em
Pode-se concluir, de acordo com a reportagem, que a
pizzas.
população da França, em 1996, em milhões de
c) Em qual das opções do item anterior os dois
habitantes, era de, aproximadamente:
comeriam mais pizzas?
A) 12,6 D) 75,7
B) 25,8 E) 103,20
106. (UENF-UERJ-01) Um atleta está treinando em
C) 51,6
uma pista retilínea e o gráfico abaixo apresenta dados
sobre seu movimento.
v (m/s)

GABARITO
2
71-200.000 pessoas; 72-D; 73-40 m2; 74- a) 600 m2,
b) 16 m; 75-E; 76- x = 1 ou x = 2; 77-R$ 32,00; 78-C;
10 79-E; 80-B; 81-C; 82-D; 83- a) A opção que Raquel
5 t (s)
sugeriu.
A distância percorrida pelo corredor, no intervalo entre b) Três maneiras diferentes: 3 grandes e 1 pequena, 2
0 e 5 segundos, é igual à área do trapézio sombreado. grandes e 4 pequenas e 1 grande e 7 pequenas. c) Na
Calcule essa distância. opção: 3 grandes e 1 pequena; 84-22,5; 85- B; 86-C

107. (UERJ-2011-1ºEQ) A embalagem de papelão de


um determinado chocolate, representada na figura
abaixo, tem a forma de um prisma pentagonal reto de
altura igual a 5 cm.

33
PROF ANDRÉ FONSECA GEOMETRIA

Para isso, tomemos como ângulo de referência o


TRIGONOMETRIA ângulo .
Para , temos:

TRIGONOMETRIA NO TRIÂNGULO RETÂNGULO


= seno de 
A palavra trigonometria, de origem grega (tri =
três, gono = ângulo, metria = medida), significa “medida de
triângulos”. Como o nome sugere, a Trigonometria é a parte
da Matemática que estuda as relações entre as medidas dos Seno de um ângulo agudo é a razão entre a medida
lados e as medidas dos ângulos de um triângulo. do cateto oposto ao ângulo e a medida da
Embora não se tenha informações precisas sobre a hipotenusa. Indica-se: sen 
origem desse estudo, há registros de sua aplicação por
babilônicos e antigos egípcios, especialmente na
Agrimensura e na Astronomia. A Trigonometria era usada,
= cosseno de 
por exemplo, para determinar distâncias que não podiam ser
medidas com instrumentos, como distâncias entre planetas.
Entre os gregos, suas primeiras aplicações práticas Cosseno de um ângulo agudo é a razão entre a
ocorreram com Ptolemaios, por volta do ano 150 d.C., que a medida do cateto adjacente ao ângulo e a medida da
usou para determinar a latitude e longitude de cidades e de hipotenusa. Indica-se: cos 
outros pontos geográficos em seus mapas.
Do mundo grego, a Trigonometria passou para a
Índia, onde era usada, a partir do século V, nos cálculos
astrológicos. No ano 800, aproximadamente, ela chega ao
= tangente de 
mundo islâmico, onde foi muito desenvolvida e aplicada na
Astronomia e Cartografia. Alcança, com os livros de
Ptolemaios, a Europa Cristã em torno do ano 1100. Com os
portugueses encontra uma aplicação de enorme valor Tangente de um ângulo agudo é a razão entre a
econômico na Navegação Oceânica. medida do cateto oposto ao ângulo e a medida do
Até cerca de 1600, todas as aplicações da cateto adjacente ao ângulo. Indica-se: tg 
Trigonometria (Astronomia, Cartografia e Navegação
Oceânica) nada tinham a ver com problemas de agrimensura VALORES DAS RAZÕES TRIGONOMÉTRICAS DOS
ou topografia. É importante observar que, nesse período, a ÂNGULOS NOTÁVEIS (30º, 45º E 60º)
Trigonometria estava num estágio bastante desenvolvido,
em muito ultrapassando o que é hoje ensinado no ensino 30º 45º 60º
médio.
Seno
A Matemática apresenta invenções tão sutis que poderão
servir não só para satisfazer os curiosos como, também para Cosseno
auxiliar as artes e poupar trabalho aos homens. (Descartes)

Tangente 1
RAZÕES TRIGONOMÉTRICAS NO TRIÂNGULO RETÂNGULO

Observe o triângulo retângulo ABC, indicado a seguir. Exercícios resolvidos:


Calcular seno, cosseno e tangente de cada ângulo
B agudo na figura.
 < 90º
 < 90º  10
 a 6
c  +  = 90º

 8
C
A b
sen = sen =
No triangulo, temos:
cos = cos =
BC - hipotenusa de medida a.
AB e AC - catetos de medidas, respectivamente c e b.
tg = tg =
Chamamos de razões trigonométricas as divisões
entre os lados do triângulo retângulo. Calcular a medida x no triangulo retângulo a seguir.
34
PROF ANDRÉ FONSECA GEOMETRIA

c) x =
12
4. (FUVEST-SP) Calcule x indicado na figura.
º
x
Na figura acima temos, para o ângulo dado (30º), o x
cateto adjacente e a hipotenusa do triângulo. 30º 60º
Das razões trigonométricas apresentadas, a única que
trabalha com esses elementos é a razão cosseno.
Portanto, vamos trabalhar com o cosseno de 30º, que 100m

na tabela trigonométrica nos dá o valor . 5. (UERJ) Um barco navega na direção AB, próximo a
um farol P, conforme a figura abaixo.
Para resolver um problema desse tipo, devemos
igualar o valor “numérico” do cosseno dado na tabela
com o valor algébrico do cosseno representado pela
figura.
Logo, teremos:

Resolvendo essa regra de três, obtemos x=6 .


No ponto A, o navegador verifica que a reta AP, da
embarcação ao farol, forma um ângulo de 30º com a
direção AB. Após a embarcação percorrer 1.000 m, no
EXERCÍCIOS ponto B, o navegador verifica que a reta BP, da
embarcação ao farol, forma um ângulo de 60º com a
1. (UNIRIO-2005) Ao ser indagado sobre o valor de mesma direção AB. Seguindo sempre a direção AB, a
sen 45º, um estudante pensou assim: menor distância entre a embarcação e o farol será
equivalente, em metros, a:
a) 500 b) 5003 c) 1.000 d) 1.0003
45º =
6. (UERJ - 2000) Observe a bicicleta e a tabela
sen 45º = trigonométrica.

Continuando nesse raciocínio, o estudante encontrou


como resposta:
a) um valor menor que o correto, diferente da metade
do correto.
b) o valor correto.
c) a metade do valor correto.
d) o dobro do valor correto. ÂNGULO
e) um valor maior que o correto, diferente do dobro do SENO COSSENO TANGENTE
(em graus)
correto. 10 0,174 0,985 0,176
11 0,191 0,982 0,194
12 0,208 0,978 0,213
2. Um avião levanta vôo em B, e sobe fazendo um
13 0,225 0,974 0,231
ângulo constante de 15º com a horizontal. A que altura 14 0,242 0,970 0,249
estará e qual a distância percorrida quando passar
pela vertical que passa por uma igreja situada a 2 km Os centros das rodas estão a uma distância PQ igual
do ponto de partida? a 120 cm e os raios PA e QB medem,
Dados: sen 15º = 0,26 e tg 15º = 0,27. respectivamente, 25 cm e 52 cm.
De acordo com a tabela, o ângulo AÔP tem o seguinte
3. (Fumec-MG) x = implica: valor:
a) 10º b) 12º c) 13º d) 14º

a) x = d) x =
7. (UNI-RIO - RJ) Um disco voador é avistado, numa
região plana, a uma certa altitude, parado no ar. Em
b) x = e) x = certo instante, algo se desprende da nave e cai em

35
PROF ANDRÉ FONSECA GEOMETRIA

queda livre, conforme mostra a figura. A que altitude


se encontra esse disco voador? 10. (UFRJ-99-2ª fase) Na figura a seguir, os círculos
de centros O1 e O2 são tangentes em B e têm raios
1cm e 3cm.

d
Considere as afirmativas:
I. A distância d é conhecida.
II. A medida do ângulo  e a tg do mesmo ângulo são conhecidas.
Então, tem-se que:
a) A I sozinha é suficiente para responder à pergunta, mas a II
sozinha, não. Determine o comprimento da curva ABC.
b) A II sozinha é suficiente para responder à pergunta, mas a I,
sozinha, não.
c) I e II, juntas são suficientes para responder à pergunta, mas GABARITO
nenhuma delas, sozinha, não é. 1- ; 2- altura: h = 0,54 km ; distância: d  2,07 km; 3-B;
d) Ambas são, sozinhas, suficientes para responder à pergunta. 4-x = 503 m; 5-B; 6-C; 7-C; 8-35,70 m; 9-D; 10-5/3
e) A pergunta não pode ser respondida por falta de dados.

8. Um observador, com 1,70m de altura, vê uma luz


no alto de uma torre de televisão, sob um ângulo de
60º, como mostra a figura. Esse observador se
encontra a 20m da base da torre (distância horizontal).
Determine a altura aproximada da torre.
(Dado: )

9. (UERJ-09) Um piso plano é revestido de


hexágonos regulares congruentes cujo lado mede 10
cm.
Na ilustração de parte desse piso, T, M e F são
vértices comuns a três hexágonos e representam os
pontos nos quais se encontram, respectivamente, um
torrão de açúcar, uma mosca e uma formiga.

Ao perceber o açúcar, os dois insetos partem no


mesmo instante, com velocidades constantes, para
alcançá-lo. Admita que a mosca leve 10 segundos
para atingir o ponto T. Despreze o espaçamento entre
os hexágonos e as dimensões dos animais.
A menor velocidade, em centímetros por segundo,
necessária para que a formiga chegue ao ponto T no
mesmo instante em que a mosca, é igual a:
(A) 3,5 (B) 5,0 (C) 5,5 (D) 7,0
A C i r c u n f e r ê n c36 i a e s e u C o m p r i m e n
to
PROF ANDRÉ FONSECA GEOMETRIA

ENTENDENDO O QUE É UMA CIRCUNFERÊNCIA COMPRIMENTO DE UMA CIRCUNFERÊNCIA

Observe a figura a seguir. Neste tópico, vamos desenvolver uma atividade e


para isso você vai precisar de:
E
 algum material que seja redondo – um
I CD, uma bandeja, uma roda etc;
O  um barbante ou um pedaço de linha
(também pode ser uma fita métrica);
 uma régua (caso não tenha uma fita
métrica)
 um lápis ou uma caneta, ou algo que
A figura apresenta alguns pontos, O, I, E e outros possa marcar um ponto na linha (ou barbante) que
pontos que não foram nomeados. estiver usando.
Com o auxílio de uma régua, meça a distância,
em cm, entre o ponto O e os pontos que não foram De posse dos materiais acima, encontre:
nomeados. Depois, coloque os resultados na tabela a
seguir. a) a medida do comprimento da borda
Distância entre Valor encontrado (circunferência) do objeto redondo;
b) a medida do diâmetro desse mesmo objeto.
OeI
OeE
Anote os valores na tabela a seguir.
O e os demais pontos
Comprimento da borda
Agora, pense e responda: Diâmetro do objeto
do objeto
Quantos outros pontos você
acha que existem de modo que a
distância entre o ponto O e cada
um desses pontos seja sempre Agora, escreva no quadro a seguir o resultado da
igual à descrita na 3ª linha da divisão (razão) entre o comprimento da borda
(circunferência) e o diâmetro do objeto, e em
tabela acima? seguida, compare o resultado encontrado com o
resultado dos seus colegas.

É a essa infinidade de pontos que chamamos


CIRCUNFERÊNCIA, ou seja, circunferência é na
verdade um conjunto de pontos tal que a distância
entre eles e um ponto fixo – neste caso, o centro – é
sempre a mesma.
E mais, a essa distância que é sempre constante
(isso quer dizer que não há variação) chamamos RAIO Saiba que as razões que vocês encontraram
da circunferência. devem ter valores próximos ao de um número muito
importante da Matemática, o qual é representado
Na figura ao lado, tem-se que pelo símbolo , que é a letra p do alfabeto grego e
o segmento OB é um raio da pronuncia-se “pi”. Observe o valor de  a seguir com
circunferência. O segmento 100 casas decimais:
OC também é um raio. 3,14159265358979323846264338327950288419716
C 9399375105820974944592307816406286208998628034
O B Repare que neste desenho o
interior da circunferência foi 825342117068...
sombreado (hachurado). Saiba também que esse número não é um número
racional, ou seja, não pode ser obtido por meio de
A essa região sombreada uma razão, como você calculou nessa atividade. Ele é
chamamos CÍRCULO. obtido por outros métodos matemáticos. Além disso,
Em qualquer circunferência, dois raios colineares  possui infinitas casas decimais.
são chamados de DIÂMETRO. Na figura acima, CB é O importante é você ter observado que todos os
um diâmetro. círculos (ou circunferências) apresentam uma
característica de regularidade expressada por esse
valor chamado .
37
PROF ANDRÉ FONSECA GEOMETRIA

Dessa regularidade, podemos então escrever a medidas de LK, MN e JK são, respectivamente, 2 m, 2


seguinte equação: m e 3 m.
Para construir o arco de circunferência IJ, o
serralheiro deve utilizar uma vara de ferro com o
Onde C representa o comprimento da seguinte comprimento:
circunferência e d, o diâmetro.

E a mesma equação pode ser escrita:

Ou ainda,
Onde r representa o raio da a) b) c) d) e)
circunferência.

7. (ENEM-98) As bicicletas possuem uma corrente que


EXERCÍCIOS liga uma coroa dentada dianteira, movimentada pelos
pedais, a uma coroa localizada no eixo da roda
1. As rodas de uma bicicleta têm 60 cm de diâmetro. traseira, como mostra a figura.
a) qual o comprimento da circunferência dessa roda? O número de voltas dadas pela roda traseira a cada
b) quantas voltas dará cada roda num percurso de pedalada depende do tamanho relativo destas coroas.
94,2 m? (apenas no item b, use  = 3,14)

2. Um ciclista dá 500 pedaladas completas numa


bicicleta cujo raio da roda é 25 cm. Considerando que
a roda dá uma volta completa a cada pedalada dada Em que opção abaixo a roda traseira dá o maior
pelo ciclista, determine quantos metros ele percorreu número de voltas por pedalada?
aproximadamente.

3. Numa circunferência de raio r = 30 cm, qual é o


comprimento de um arco que subentende um ângulo
central de 60º?
Considere  = 3,14

4. Determine o perímetro da figura sombreada,


indicada a seguir.
A M D

8. (ENEM-98 - Adaptada) Quando se dá uma pedalada


na bicicleta a seguir (isto é, quando a coroa acionada
B N C
pelos pedais dá uma volta completa), qual é a
distância aproximada percorrida pela bicicleta, em
Sabe-se que ABCD é um quadrado de lado 10 cm. As metros?
linhas curvas são semicircunferências com centros nos (use  = 3)
pontos médios, M e N, dos lados do quadrado.

5. Supondo que a Terra seja uma esfera de raio r =


6.375 km, determine a distância do equador a um
ponto situado a uma latitude 30º norte.
Adote  = 3,14.

6. (UFF) Um serralheiro deseja construir a grade de


GABARITO:
ferro desenhada abaixo.
1-a) 60 cm, b) 50 voltas; 2 – 785 m; 3- 31,4 cm; 4-10(2 + ) cm; 5
Sabe-se que MN é mediatriz do lado LK do retângulo - 3.336,25 km; 6-A; 7-A; 8-7,2 m;
IJKL, a medida do ângulo JMK = IML = 60º e as

38
PROF ANDRÉ FONSECA GEOMETRIA

II II
TRIGONOMETRIA I

I
 2
III IV III IV
TRIGONOMETRIA NO CICLO
COMPRIMENTO DE CIRCUNFERÊNCIA 270º 3/2
Supondo que pudéssemos cortar uma Podemos, a partir da informação 180º =  e de uma regra de
circunferência e esticá-la, retificando-a, obteríamos um três simples, calcular o valor de qualquer arco em graus ou
segmento de reta AB. O comprimento desse segmento é em radianos.
denominado comprimento da circunferência. Exemplo: Qual é o arco em radianos, equivalente ao arco
de 150º?
Solução:
r Valor do arco em graus Valor do arco em rad
180º 
150º x

A circunferência retificada B

A medida do comprimento da circunferência é dada


por:

C = r
Aqui, o valor de p é 3,1415926... ARCO ORIENTADO B
A figura mostra que o
UNIDADES DE MEDIDA DE ARCOS E ÂNGULOS percurso de A para B pode ser feito
Para medir arcos e ângulos utilizam-se duas no sentido anti-horário, seguindo o
medidas: o grau e o radiano. menor arco AB, ou no sentido O A
horário, seguindo o maior arco AB.
O Grau
Dividindo uma circunferência em 360 partes iguais, Estabelecendo como positivo o
cada uma dessas partes é um arco de 1º. (lê-se: um grau) sentido anti-horário (contrário ao movimento dos ponteiros
do relógio) e como negativo o sentido horário, temos:
O Radiano B B sentido
Um radiano é o ângulo central de um círculo que positivo
subentende um arco de comprimento igual ao raio do
círculo.
O A O A
A
r sentido
arco AB de comprimento r negativo
1 rad
r
B

Neste caso, como o comprimento do raio é igual ao


comprimento arco, a medida do ângulo central é, em CIRCUNFERÊNCIA TRIGONOMÉTRICA OU
radianos, igual a 1. CICLO TRIGONOMÉTRICO
Como o comprimento de uma circunferência pode ser É a circunferência de raio unitário (r = 1) que se
calculado pela expressão 2r, podemos dizer quantas vezes divide em quatro regiões, por dois diâmetros
esse arco AB cabe na circunferência através da divisão: perpendiculares. Cada região é um quadrante.
y
, ou seja, 2 vezes.
B(0, 1)
Portanto, podemos concluir que o ângulo central
correspondente a uma volta é, em radianos, 2 rad.
Podemos, a partir dessa informação, estabelecer uma relação C(- 1, 0) r = 1 A(1, 0)
x
entre medidas de ângulos em graus e radianos. Algumas
delas estão representadas a seguir.
D(0, - 1)
ARCOS CÔNGRUOS
90º /2
39

180º 0º  360º 0
PROF ANDRÉ FONSECA GEOMETRIA

Considere um ponto P localizado no 1º quadrante


do ciclo trigonométrico de modo que o arco AP (A é ponto sen  =
de origem do ciclo) forme um ângulo de 60º, ou seja, /3. É
possível dar várias voltas num sentido ou noutro de maneira cos  =
que sempre o arco produzido tenha a mesma extremidade P.
Chamamos esses arcos de arcos côngruos.
Lembre-se: uma volta é equivalente a 360º ou 2 rad. Daí, definimos:
Observe. y  sen  é a ordenada do ponto P;
  cos  é a abscissa do ponto P.
P
3
O eixo y é o eixo dos senos e o eixo x é o eixo dos cossenos.
60º A Se P é um ponto do ciclo trigonométrico, podemos escrever:
O x M(cos, sen).
Essa nova ideia nos permite ir além dos ângulos agudos; e
passamos a ter também valores de senos, cossenos e
tangentes tanto positivos quanto negativos, dependendo do
Existem infinitos arcos com extremidade em P, ou seja, quadrante onde se encontra a extremidade do arco estudado.
A tabela a seguir fornece os sinais das razões seno, cosseno
côngruo a 60º ou . e tangente de acordo com os quadrantes.
1º Quad 2º Quad 3º Quad 4º Quad
60º (60º + 0  360º) 60º (60º + 0  360º) seno + + - -
420º (60º + 1  360º) – 300º (60º – 1  360º) cosseno + - - +
780º (60º + 2  360º) – 660º (60º – 2  360º) tangente + - + -
1.140º (60º + 3  360º) – 1.020º (60º – 3  360º)

Importante: como a circunferência trigonométrica possui


De um modo geral, podemos escrever uma expressão que
raio unitário, ou seja, igual a 1, o seno ou o cosseno de
represente todos os arcos côngruos a 60º:
qualquer arco nunca pode ser menor que – 1 ou maior que
1.
60º + 360ºk, k 
- 1  sen   1
Exemplo: - 1  cos   1
Determinar todos os arcos côngruos ao arco de A tangente pode ser calculada como a razão entre o seno e
o cosseno e o seu valor pode ser qualquer número real.
medida .
Aplicando o Teorema de Pitágoras ao triangulo retângulo
Dando voltas no sentido positivo, temos: OPB da última figura, obtemos:
, , ... P

Ou no sentido negativo:
, , ...
O B
De modo geral, podemos escrever que todos os arcos
côngruos a são arcos do tipo EXERCÍCIOS

, k . 9. As rodas de uma bicicleta têm 60 cm de diâmetro.


a) qual o comprimento da circunferência dessa roda?
b) quantas voltas dará cada roda num percurso de 94,2 m?
SENO E COSSENO NO CICLO
Use  = 3,14.
Seja P a extremidade do arco AP no primeiro
quadrante, como mostra a figura a seguir.
10.Numa circunferência de raio r = 30 cm, qual é o
Lembre-se: no ciclo trigonométrico, o raio vale 1.
y comprimento de um arco que subentende um ângulo central
Ade 60º? M D
P A(1, 0); P(B, C) Considere  = 3,14
C
1
 A
O B x
11.Determine o perímetro da figura sombreada, indicada a
seguir.
Do triângulo retângulo OPB, temos:
B N C
40
PROF ANDRÉ FONSECA GEOMETRIA

Sabe-se que ABCD é um quadrado de lado 10 cm. As linhas a) b) c) d) e)


curvas são semicircunferências com centros nos pontos
médios, M e N, dos lados do quadrado.
Considere  = 3,14.
16.(ENEM-98) As bicicletas possuem uma corrente que liga
12.Supondo que a Terra seja uma esfera de raio r = 6.375 uma coroa dentada dianteira, movimentada pelos pedais, a
km, determine a distância do equador a um ponto situado a uma coroa localizada no eixo da roda traseira, como mostra
uma latitude 30º norte. a figura.
Adote  = 3,14. O número de voltas dadas pela roda traseira a cada pedalada
depende do tamanho relativo destas coroas.
13.(UNICAMP) Os pontos A e B estão, ambos, localizados
na superfície terrestre a 60º de latitude norte; o ponto A está
a 15º 45’ de longitude leste e o ponto B a 56º 15’ de
longitude oeste.
a) Dado que o raio da Terra, considerada perfeitamente
Em que opção abaixo a roda traseira dá o maior número de
esférica, mede 6.400 km, qual é o raio do paralelo de 60º?
voltas por pedalada?
b) Qual é a menor distância entre os pontos A e B, medida
ao longo do paralelo de 60º?
(use 22/7 como aproximação para )

14.(UERJ-2005-II) A Terra pode ser representada por uma


esfera cujo raio mede 6.400 km. Na representação abaixo,
está indicado o trajeto de um navio do ponto A ao ponto C,
passando por B.

Qualquer ponto da
superfície da Terra tem
coordenadas (x;y), em que
x representa a longitude e
y, a latitude. As
coordenadas dos pontos A,
B e C estão indicadas na
tabela a seguir.
17.(ENEM-98) Quando se dá uma pedalada na bicicleta ao
lado (isto é, quando a coroa acionada pelos pedais dá uma
Coordenadas
Pontos volta completa), qual é a distância aproximada percorrida
x y pela bicicleta, sabendo-se que o comprimento de um círculo
A 135º 0º de raio R é igual a 2R, onde   3?
Considerando  igual a 3, a B 135º 60º
distância mínima, em C 90º 60º
quilômetros, a ser percorrida
pelo navio no trajeto ABC é igual a:
a) 11.200 b) 10.800 c) 8.800 d) 5.600

15.(UFF) Um serralheiro deseja construir a grade de ferro


desenhada abaixo.
Sabe-se que MN é mediatriz do lado LK do retângulo IJKL 18. (PUC) O valor de sen 1200º é igual a:
e as medidas de LK, MN e JK são, respectivamente, 2 m, 2 a) cos 60º d) – sen 30º
m e 3 m. b) – sen 60º e) cos 45º
Para construir o arco de circunferência IJ, o serralheiro deve c) cos 30º
utilizar uma vara de ferro com o seguinte comprimento:

19. (UFF-97) Para  = 89º, conclui-se que:


a) tg  < sen  < cos 
b) cos  < sen  < tg 
41
PROF ANDRÉ FONSECA GEOMETRIA

c) sen  < cos  < tg 


d) cos  < tg  < sen  C; 11-B; 12-D; 13-D; 14-D; 15-D; 16- = ;= ;=
e) sen  < tg  < sen 
;= ; 17-E; 18-D.
20. (UFF-2002) Seja x um arco do primeiro quadrante tal
que sen x = 0,6. Pode-se afirmar que:

a) sen = - 0,3 d) cos x = 0,8

b) cos 2x = - 0,6 e) sen 2x = 1,2

c) cos = 0,6

21. (CESGRANRIO) O número de soluções da equação


sen2 x = 2 sen x, no intervalo [ 0, 2 ], é:
a) 0 d) 3
b) 1 e) 4
c) 2

22. No intervalo [ 0 , 2 ], o número de soluções distintas


da equação é:
a) 0 d) 3
b) 1 e) 4
c) 2

23. (UGV - 78) Sejam sen x = e cos x = .


Então, o valor de a é:
a) 1 ou 0 b) -1 ou 0 c) 2 ou - 2 d) 0 e) -1

24. (UFRJ - 2004 - 2ª fase) A equação


x2 - 2x cos θ + sen2 θ = 0 possui raízes reais iguais.
Determine θ, 0  θ  2.

25. (UFF-00) Considere os ângulos ,  e , conforme


representados no círculo.
Pode-se afirmar que:
a) cos  < cos 
b) sen  < cos 
c) cos  > cos 
d) cos  < cos 
e) sen  > sen 

26. (UERJ - 2003) Observe a matriz a seguir.

Resolvendo seu determinante, será obtido o seguinte


resultado:
a) 1 b) sen x
c) sen2 x d) sen3 x

GABARITO:
1-a) 1,884m, b) 50 voltas; 2- 31,4; 3-51,4 cm; 4-3.336,25
km; 5-a) 3.200km, b) 4.022,86km; 6-C; 7-A; 8-A; 9-C; 10-
42
PROF ANDRÉ FONSECA GEOMETRIA

 triangulares – quando as bases são triângulos,


 quadrangulares – quando as bases são quadriláteros,
 pentagonais – quando as bases são pentágonos,
 hexagonais – quando as bases são hexágonos, etc.

De acordo com a inclinação (ângulo) entre as faces


SÓLIDOS GEOMÉTRICOS laterais e as bases, os prismas podem ser:
 retos – quando as arestas laterais são perpendiculares aos
planos das bases.
 oblíquos – quando as arestas laterais são obliquas aos planos
Prismas das bases.
IMPORTANTE:
Considere um polígono contido num plano . Considere Quando um prisma é reto e a bases são polígonos regulares, ele é
também uma reta que intercepta este plano. Observe a figura. denominado Prisma Regular.
r
Para cada ponto do
polígono no plano a, considere Prismas
segmentos de reta paralelos à Notáveis_____________
reta r. paralelepípedo reto ________________
r
PARALELEPÍPEDO
São prismas formados por seis faces, todas em forma de
paralelogramos.

paralelepípedo oblíquo
Considere  um plano paralelo a . Para cada ponto do
polígono em , existe um segmento paralelo à reta r com
extremidades nos dois planos.
CUBO
r
Ao conjunto formado por todos Paralelepípedo reto formado por arestas de
os segmentos paralelos à reta r mesma medida.
com extremidades nos planos 
e (sólido geométrico),
d
denominamos prisma.

ELEMENTOS c
D
DIAGONAL DE UM PARALELEPÍPEDO
face RETÂNGULO (D)
b

aresta lateral
a Considere o paralelepípedo ao lado.
Chamaremos por a o seu comprimento, b a sua largura e c a sua
h altura.
face lateral
base
bases: são os O comprimento da diagonal D pode ser calculado aplicando o
polígonos aresta da base teorema de Pitágoras no triangulo retângulo de medidas D, d e c,
congruentes e onde D é a diagonal do paralelepípedo, d é a diagonal do
paralelos. retângulo da base e c é a altura.
faces laterais: são os paralelogramos. (I)
arestas das bases: são os lados dos polígonos das bases. Aplicando Pitágoras ao triângulo retângulo da base, temos:
arestas laterais: são os lados do prisma que ligam uma base a
(II)
outra. São paralelas e de comprimentos congruentes.
altura: é a distância entre os planos das bases. (II) em (I) 

CLASSIFICAÇÃO

De acordo com os polígonos das bases, os prismas


podem ser: Para o cubo, temos que sua diagonal D mede:

43
PROF ANDRÉ FONSECA GEOMETRIA

4. (UNI-RIO) Os lados de um cubo são multiplicados


por 2. O volume do cubo fica:
a) 2 vezes maior
onde a é a medida da aresta. b) 3 vezes maior
c) 4 vezes maior
d) 6 vezes maior
ÁREA TOTAL DA SUPERFÍCIE DE UM PARALELEPÍPEDO RETÂNGULO (St) e) 8 vezes maior
Calcular a área da superfície de um paralelepípedo retângulo é o
mesmo que calcular a soma das áreas das faces retangulares. 5. (UFPE) Segundo o regulamento de uma companhia
de transporte, a bagagem de mão de um
passageiro, na forma de um paralelepípedo reto,
deve ter altura de, no máximo, 45 cm e a soma da
largura e do comprimento não pode ultrapassar 80
cm.
Para qual valor da largura, medida em cm, o volume da
bagagem de mão será o máximo?
Para o cubo, temos:
6. (UERJ) Na construção de um hangar, com a forma
de um paralelepípedo retângulo, que possa abrigar
um “Airbus” foram consideradas as medidas
apresentadas abaixo:

VOLUME (V)
De modo geral, o volume de qualquer prisma pode ser calculado
multiplicando a área da base do prisma pela sua altura.
Para o paralelepípedo acima, temos que o seu volume é dado por:

onde a, b e c são, respectivamente, o comprimento, a largura e a 79,8 metros


altura do paralelepípedo.

Para o cubo, temos:

24,1 metros

onde a é a aresta do cubo.

EXERCÍCIOS 73 metros
(Adaptado de Veja, 14/06/2000)
1. (UFRJ-03) Uma pedra de massa 25 kg tem a forma
de um paralelepípedo com 2 cm de espessura. Sua Calcule o volume mínimo desse hangar.
base é um quadrado com 1 m de lado. Qual a
massa de uma outra pedra, do mesmo material, 7. Um caminhão basculante tem a carroceria com as
que tem a forma de um paralelepípedo com 2 m dimensões indicadas na figura.
de comprimento, 80 cm de
largura e 3 cm de espessura?

2. (UNIRIO-05) Uma sala de 8 m de comprimento, 60


dm de largura e 30 dm de altura deverá ser
ocupada por 48 pessoas. Sabe-se que a
quantidade de ar necessária para que uma pessoa
tenha boas condições de permanecer numa sala é
4 m3 .
De quanto, no mínimo, se deve aumentar a medida da
altura dessa sala, para que a necessidade de ar de todas as
pessoas que lá estarão seja plenamente satisfeita?
a) 2m b) 1,5m c) 1m d) 0,75m e) 0,50m Calcule quantas viagens deverá fazer para transportar
136m3 de areia.
3. (UFF-01) Uma piscina tem a forma de um prisma
reto, cuja base é um retângulo de dimensões 15 m
8. Depositam-se 5.000 litros de água no interior de um
e 10 m. reservatório com a forma de um paralelepípedo
A quantidade necessária de litros de água para que o nível reto-retângulo de 10 m de comprimento por 5 m de
de água da piscina suba 10 cm é: largura. A partir desse instante, um vazamento
a) 0,15 L b) 1,5 L c) 150 L d) 1.500 L e) 15.000 L começa a baixar o nível d’água, à base de um
centímetro a cada 24 horas.

44
PROF ANDRÉ FONSECA GEOMETRIA

O número de horas que, no mínimo, deverão transcorrer reta r) com extremidades no plano  e em um plano , paralelo a
para que o reservatório fique completamente vazio .
(desprezando-se qualquer outra eventual forma de perda
d’água) é:
a) 96 b) 120 c) 144 d) 192 e) 240
r
Ao conjunto formado
9. (UERJ-2012-2ºEQ) As figuras a seguir mostram
 por todos os segmentos
dois pacotes de café em pó que têm a forma de paralelos a r com extremidades
paralelepípedos retângulos semelhantes. nos planos  e , chama-se
cilindro.

ELEMENTOS

altura h
raio da base
base

geratriz

eixo
base
Se o volume do pacote maior é o dobro do volume do menor, a
razão entre a medida da área total do maior pacote e a do menor é bases: são os círculos contidos nos planos  e ..
igual a:
altura: é a distância entre as bases.
a) eixo: é a reta que passa pelo centro das bases.
b) geratriz: é qualquer segmento paralelo ao eixo, com extremidades
nos pontos das circunferências das bases.
c)
d)
CLASSIFICAÇÃO

Um cilindro circular pode ser reto ou oblíquo.

 reto – quando as geratrizes são perpendiculares as bases.


geratriz

altura
=

GABARITO
1-60kg; 2-C; 3-E; 4-E; 5-40cm; 6-140.392,14 m; 7-20; 8-E.
9-B;
Nota: Quando num cilindro circular reto, a medida da altura é igual
à medida do diâmetro do círculo da base, o cilindro é denominado
cilindro equilátero.
Cilindros
 oblíquo – quando as geratrizes são oblíquas as bases.
Considere um plano  e um círculo contido neste plano.
Considere também uma reta r que intercepta este plano. Observe
a figura. geratriz altura
r
Considere também
 segmentos de retas (paralelos a
IMPORTANTE:

45
PROF ANDRÉ FONSECA GEOMETRIA

Um cilindro circular reto pode ser obtido pela rotação completa de I - Dá-se uma volta completa em torno do tronco com um
um retângulo em torno de um eixo. Por esse fato, dizemos que barbante.
esse tipo de cilindro é um sólido de revolução.

geratriz

II - O barbante é dobrado duas vezes pela ponta e, em


seguida, seu comprimento é medido com fita métrica.
raio

1ª dobra 2ª dobra
VOLUME (V)
Como a ideia da construção do (sólido) cilindro é III - O valor obtido com essa medida é multiplicado por ele
análoga à construção de um prisma, isto é, uma região localizada mesmo e depois multiplicado pelo comprimento do tronco.
entre dois planos paralelos, o volume do cilindro será análogo ao Esse é o volume estimado de madeira.
volume do prisma, ou seja, o produto entre a área da base e a
Outra estimativa pode ser obtida pelo cálculo formal do
altura. Como a base é um círculo, teremos: volume do tronco, considerando-o um cilindro perfeito.
A diferença entre essas medidas é praticamente equivalente
às perdas de madeira no processo de corte para
comercialização.
onde r é o raio da base e h é a altura do cilindro. Pode-se afirmar que essas perdas são da ordem de
A) 30% B) 22% C) 15% D) 12% E) 5%
ÁREA TOTAL DA SUPERFÍCIE DO CILINDRO (ST)
A área total da superfície de um cilindro será a soma das 4. (ENEM-03) Uma editora pretende despachar um lote de
livros, agrupados em 100 pacotes de 20 cm x 20 cm x 30
áreas das bases com a área lateral. Se pudéssemos retirar as
cm. A transportadora acondicionará esses pacotes em
“cascas” de um cilindro, teríamos dois círculos (bases) e um caixas com formato de bloco retangular de 40 cm x 40 cm x
retângulo (área lateral). 60 cm. A quantidade mínima necessária de caixas para esse
envio é:
A) 9 B) 11 C) 13 D) 15 E) 17

5. (UERJ-01) Um recipiente cilíndrico de 60 cm de altura e


Portanto, a área total da superfície de um cilindro (ST) deve ser: base com 20 cm de raio está sobre uma superfície plana
horizontal e contém água até a altura de 40 cm, conforme
indicado na figura.
Imergindo-se totalmente um bloco 20cm
cúbico no recipiente, o nível da água
sobe 25%.
Considerando  igual a 3, a medida,
60cm
em cm, da aresta do cubo colocado 40cm
na água é igual a:
EXERCÍCIOS A) B)
C) D)
1. (ENEM-99) Uma garrafa cilíndrica está
fechada, contendo um líquido que ocupa quase
6. (UFF-04) “Uma das soluções encontradas para a
completamente seu corpo, conforme mostra a
figura. Suponha que, para fazer medições, você escassez de água na região semi-árida do nordeste
disponha apenas de uma régua milimetrada. brasileiro é a captação da água da chuva que escorre dos
Para calcular o volume do líquido contido na telhados das casas. A água captada é conduzida por meio
garrafa, o número mínimo de medições a serem de calhas para um reservatório com a forma de um cilindro
realizadas é: circular reto.”
Superinteressante, Edição 177, junho de 2002.
A) 1 B) 2 C) 3 D) 4 E) 5
O reservatório citado tem altura aproximada de 1,8 metro e
capacidade para armazenar 16000 litros da água da chuva.
2. (ENEM-99) Para calcular a capacidade total da garrafa, Considerando R o raio da base do reservatório, pode-se
lembrando que você pode virá-la, o número mínimo de afirmar que R2, em metro quadrado, é aproximadamente:
medições a serem realizadas é: a) 1,4 b) 1,9 c) 2,8 d) 3,8 e) 7,8
A) 1 B) 2 C) 3 D) 4 E) 5
7. (FUVEST-04) Uma metalúrgica fabrica barris cilíndricos
3. (ENEM-01) Em muitas regiões do Estado do de dois tipos, A e B, cujas superfícies laterais são moldadas
Amazonas, o volume de madeira de uma árvore cortada é a partir de chapas metálicas retangulares de lados a e 2a,
avaliado de acordo com uma prática dessas regiões: soldando lados opostos dessas chapas, conforme ilustrado
ao lado.

46
PROF ANDRÉ FONSECA GEOMETRIA

Barril do TIPO A a a
2a 

A figura geométrica formada pela reunião de todos


Barril do TIPO B 2a os segmentos de reta que têm uma extremidade no ST = Sl + Sb
2a
ponto V e a outra num ponto do polígono P
a denomina-se pirâmide.
Se VA e VB indicam os volumes dos barris do tipo A e B,
respectivamente, tem-se:
a) VA = 2VB ELEMENTOS
b) VB = 2VA
c) VA = VB V
vértice
d) VA = 4VB
e) VB = 4VA aresta lateral
altura
face lateral
8. (UFF-01) Um reservatório, na forma de um cilindro
circular reto, tem raio da base r, altura h e volume V.
Deseja-se construir outro reservatório que tenha, também, a 
forma de um cilindro circular reto, volume V, porém, raio da
base igual a r/2r e altura H.
A relação entre as alturas desses reservatórios é dada por: aresta da base base

9. (UFRJ) Mário e Paulo possuem piscinas em suas base: é o polígono convexo situado no plano .
casas. Ambas têm a mesma profundidade e bases com o vértice: é o “ponto de encontro” das arestas.
mesmo perímetro. A piscina de Mário é um cilindro circular
faces laterais: são os triângulos.
reto e a de Paulo é um prisma reto de base quadrada. A
companhia de água da cidade cobra R$1,00 por metro arestas das bases: são os lados do polígono da base.
cúbico de água consumida. arestas laterais: são segmentos que ligam o vértice da
a) Determine qual dos dois pagará mais para encher de pirâmide aos vértices do polígono da base.
água a sua piscina. altura: distância entre o vértice da pirâmide e o plano .
b) Atendendo a um pedido da família, Mário resolve duplicar
o perímetro da base e a profundidade de sua piscina,
mantendo, porém, a forma circular. As pirâmides são classificadas de acordo com o polígono
Determine quanto Mário pagará pela água para encher a da base, ou seja, podem ser:
nova piscina, sabendo que anteriormente ele gastava RS  triangulares – quando a base é um triângulo.
50,00.  quadrangulares – quando a base é um quadrilátero.
 pentagonais – quando a base é um pentágono. Etc.
10. (RURAL-RJ) Carlos é um rapaz viciado em beber
refrigerante diet. Um dia, voltando do trabalho, ele passou
em frente a uma companhia de gás, onde viu um enorme
reservatório cilíndrico de 3 metros de altura e 2 metros de IMPORTANTE:
diâmetro e pensou... “Em quanto tempo eu beberia aquele
reservatório inteiro, se ele estivesse cheio de refrigerante
diet?” Uma pirâmide é regular quando sua base é um polígono
Considerando  = 3,14 e sabendo-se que Carlos bebe 3 regular e a projeção ortogonal do vértice sobre o plano
litros de refrigerante diet por dia, pode-se afirmar que ele da base é o centro da base.
consumirá todo o líquido do reservatório em um período de: Numa pirâmide regular, as faces laterais são triângulos
a) 86 dias. isósceles congruentes.
b) 86 meses. A altura de cada um desses triângulos é chamada de
c) 86 anos.
d) 8,6 anos. apótema da pirâmide.
e) 860 meses.
GABARITO
1-B; 2-C; 3-B; 4-C; 5-D; 6-C; 7-A; 8-A

Pirâmides VOLUME (V)


O volume de uma pirâmide é a terça parte do
produto da área da sua base pela sua altura.
Considere um plano  e um polígono contido Seja P uma pirâmide cuja área da base é Sb e a
neste plano. Considere também um ponto V que não sua altura, h. Então, seu volume será:
pertence a ao plano .
V

47
PROF ANDRÉ FONSECA GEOMETRIA

ÁREA TOTAL DA SUPERFÍCIE DA PIRÂMIDE (ST) 2. (UNIRIO) Um prisma de altura H e uma pirâmide têm bases
A área total da superfície de uma pirâmide regular é a com a mesma área. Se o volume do prisma é a metade do volume
soma das áreas lateral e da base, ou seja: da pirâmide, a altura da pirâmide é:
a) H/6 b) H/3 c) 2H d) 3H e) 6H
onde Sl é a área lateral e Sb é a área da base.
3. (UNIRIO) Uma pirâmide é mergulhada num aquário cúbico
cheio d’água, como na figura.
Lembre-se de que a área de um triangulo é:
O número que expressa a relação entre a
quantidade de água final no aquário e a inicial
(antes de mergulhar a pirâmide) é de,
aproximadamente,
a) 25%. b) 33%. c) 50%. d) 67%. e) 72%.
VOLUME DO TRONCO DA PIRÂMIDE
Considere uma pirâmide seccionada (“cortada”) por um 4. (Transpetro-2006) A pirâmide reta, ilustrada abaixo, tem base
plano paralelo a base da pirâmide, que intercepta todas V quadrada de aresta igual a 10cm, o
as suas laterais; o sólido limitado pela base da pirâmide e ponto O como centro do quadrado, M
como ponto médio de AB, e VM =
pelo plano secante é chamado de tronco de pirâmide.
B
13cm. O volume dessa pirâmide, em
C
O M
cm3, é:
D A

a) 1 200 b) 400 c) 250 d) 4003 e) 2503


d
5. (FUVEST-2003) Um telhado tem a forma da superfície lateral de
h uma pirâmide regular, de base quadrada. O lado da base mede 8m e a
altura da pirâmide 3m. As telhas para cobrir esse telhado são vendidas
Sb em lotes que cobrem 1m2. Supondo que possa haver 10 lotes de telhas
k desperdiçadas (quebras e emendas), o número mínimo de lotes de telhas
a ser comprado é:
a) 90 b) 100 c) 110 d) 120 e) 130
SB
6. (UERJ-98) Dispondo de canudos de refrigerantes, Tiago
deseja construir pirâmides. Para as arestas laterais, usará sempre
canudos com 8 cm, 10 cm e 12 cm de comprimento. A base de
cada pirâmide será formada por 3 canudos que têm a mesma
onde medida, expressa por um número inteiro, diferente das anteriores.
h é a altura da pirâmide, Veja o modelo abaixo:
12
SB é a área da base maior do tronco da pirâmide e 8
A quantidade de pirâmides de bases
Sb é a área da base menor do tronco da pirâmide. 10
diferentes que Tiago poderá construir,
é:
IMPORTANTE: a) 10 b) 9 c) 8 d) 7
A relação entre os volumes e as alturas das
pirâmides acima é
7. (UERJ-2000) A figura abaixo representa o brinquedo Piramix.
Ele tem a forma de um tetraedro regular, com cada face dividida
em 9 triângulos equiláteros congruentes.
Se, a partir de cada vértice, for retirada
uma pirâmide regular cuja aresta é 1/3
onde V1 é o volume da pirâmide maior (pirâmide inicial), da aresta do brinquedo, restará um novo
V2 é o volume da pirâmide menor (obtida com a secção sólido.
plana), h1 é a altura da pirâmide maior e h2 é a altura da A razão entre as superfícies totais desse
pirâmide menor. sólido e do Piramix equivale a:
EXERCÍCIOS a)4/9 b) 5/9 c) 7/9 d) 8/9
8. (UERJ-02) Leia os quadrinhos.

1. (FUVEST) O número de faces triangulares de uma pirâmide é BEM, LEVEI 20 ANOS, MAS
FINALMENTE, ECONOMIZEI É PRA
11. Pode-se, então, afirmar que esta pirâmide possui: O BASTANTE PARA COMPRAR COLOCAR
ONDE?
AQUELE PEDACINHO DE
a) 33 vértices e 22 arestas. TERRA QUE QUERIA
b) 12 vértices e 11 arestas.
c) 22 vértices e 11 arestas.
d) 11 vértices e 22 arestas.
e) 12 vértices e 22 arestas.
HAGAR, o horrível Chris Browne

48
PROF ANDRÉ FONSECA GEOMETRIA

Suponha que o volume de terra acumulada no carrinho-de-mão do


personagem seja igual ao do sólido esquematizado na figura abaixo,
formado por uma
pirâmide reta
sobreposta a um
70cm paralelepípedo
retângulo. 
(O globo, março 2000)

40cm
A
60cm Um cone reto pode ser obtido pela rotação (revolução)
100cm de um triangulo retângulo em torno de um de seus
catetos. O cone é denominado cone de revolução.
Assim, o volume médio de terra que Hagar acumulou em cada ano de segmentos de reta que têm uma extremidade no ponto V
trabalho é, em dm3, igual a: e a outra num ponto do círculo C denomina-se cone
A) 12 B) 13 C) 14 D) 15 circular ou, simplesmente, cone.

9. (UFRJ-2006-1ª fase) Em um tanque no formato de um cubo de V


aresta 25 cm, contendo líquido, foi posta uma pirâmide P1, de altura igual
a 6 cm, com a base apoiada no fundo do tanque. Com isso, o nível de
líquido passou de 18 cm para 19 cm.

C

18 19
ELEMENTOS
V
vértice
a) Calcule o volume, em cm3, da pirâmide P1.
geratriz
b) A pirâmide P1 foi retirada do tanque e o nível de líquido voltou ao altura
inicial. Uma pirâmide P2, de 30 cm de altura, foi então posta no tanque, eixo
com a base apoiada no fundo, o que elevou em 2 cm o nível de líquido. O C

base

base: é o círculo C, de centro O.


vértice: é o ponto V.
altura: é a distância do ponto V ao plano da base.
eixo: é a reta que contém o vértice V e o centro do círculo
20 da base.
geratriz: é qualquer segmento que liga o vértice a
qualquer ponto da circunferência da base.

Um cone é classificado como oblíquo quando o seu eixo é


Determine o volume da pirâmide P2. oblíquo ao plano da base. Neste caso, o eixo não coincide
10. (UFF-2005) A grande pirâmide de Quéops, antiga construção
com a altura do cone.
V
localizada no Egito, é uma pirâmide regular de base quadrada, com 137
m de altura. Cada face dessa pirâmide é um triângulo isósceles cuja
altura relativa à hipotenusa mede 179 m.
A área da base dessa pirâmide, em m2 é:
a) 13.272 b) 26.544 c) 39.816 d) 53.088 e) 79.432

GABARITO:
1-E; 2-C; 3-D; 4-B; 5-A; 6-C; 7-C; 8-D; 9-a) 625 cm3, b) 16.875/13 cm3; 10-D

Cones Um cone é classificado como reto quando o seu r2 + rg


eixo é perpendicular ao plano da base. Neste caso,
Na figura a seguir, temos um plano , um círculo o eixo coincide com a altura do cone.
C não contido em  e um ponto V que não pertence a .
V V

49
PROF ANDRÉ FONSECA GEOMETRIA

2. (CESGRANRIO) No desenho a seguir, dois reservatórios de


altura H e raio R, um cilíndrico e outro cônico, estão totalmente
vazios e cada um será alimentado por uma torneira, ambas de
mesma vazão. Se o reservatório cilíndrico leva 2 horas e meia
para ficar completamente cheio, o tempo necessário para que isto
ocorra com o reservatório cônico será de:
IMPORTANTE:
altura do cone

geratriz
do cone

a) 2h b) 1h 30min c) 1h d) 50min e) 30min

3. (CESGRANRIO) Uma ampulheta é formada por dois cones


de revolução iguais, com eixos verticais e justapostos pelo vértice,
raio da o qual tem um pequeno orifício que permite a passagem de areia
base do cone da parte de cima para a parte de baixo. Ao ser colocada para
marcar um intervalo de tempo, toda a areia está na parte de cima
VOLUME (V) e, 35 minutos após, a altura da areia na parte de cima reduziu-se
O volume de um cone é a terça parte do volume à metade, como mostra a figura. Supondo que em cada minuto a
de um cilindro que tenha a mesma base a mesma altura quantidade de areia que passa do cone de cima para o de baixo é
constante, em quanto tempo mais toda a areia terá passado para
do cone.
a parte de baixo?

onde SB é a área da base do cone e h, a sua altura.

ÁREA TOTAL DA SUPERFÍCIE DO CONE (ST)


A área total da superfície de um cone é a soma da
sua área lateral com a sua base.
a) 5 minutos. b) 10 minutos. c) 15 minutos.
Ao “descascarmos” o cone, temos duas regiões
d) 20 minutos. e) 30 minutos.
planas: um círculo (base do cone) e um setor circular
(área lateral do cone). Portanto, a área total da superfície 4. (UFF-98) A figura abaixo representa um prisma obliquo de
de um cone será: bases quadradas. Na base inferior foi inscrita uma circunferência
Área da base + Área lateral C e na base superior foram traçadas as diagonais RT e SU.
Assinale a alternativa que estabelece a comparação correta entre
onde r é o raio da base do cone e g é o comprimento da sua os volumes VR, VP e VT dos três cones de base comum C e
geratriz. vértices R, P e T, respectivamente.

R
EXERCÍCIOS a) VR < VP < VT
U
P
b) VR < VP = VT
1. (UNI-RIO) Uma tulipa de chopp tem a forma cônica, como c) VR > VP > VT S T
mostra a figura a seguir. Sabendo-se que a sua capacidade é de d) VR = VP = VT
100  ml, a altura h é igual a: e) VR = VP < VT
10 cm

h 5. (UFRJ-2001-Não específica) Um recipiente em forma de cone


circular reto de altura h é colocado com vértice para baixo e com
50
PROF ANDRÉ FONSECA GEOMETRIA

eixo na vertical, como na figura. O recipiente, quando cheio até a O recipiente II tem a forma de um tronco de cone com raio da
borda, comporta 400 ml. base maior igual a 2L, raio da base menor igual a L e altura igual a
L.

h
2L

L
L

O recipiente III tem a forma de um paralelepípedo de base


Determine o volume de líquido quando o nível está em h/2. quadrada de lado igual a L e altura igual a 2L.

6. (UFF-01) Considerando um lustre de formato cônico com


altura e raio da base igual a 0,25 m, a distância do chão (H) em
que se deve pendurá-lo para obter um lugar iluminado em forma 2L
de círculo com área de 25  m2, é de

L
L

Considerando VI, VII e VIII os volumes dos recipientes I, II e III,


respectivamente, pode-se afirmar que:
a) VI  VII  VIII
b) VI  VIII  VII
c) VII  VI  VIII
d) VII  VIII  VI
e) VIII  VI  VII
a) 12 m. b) 10 m. c) 8 m. d) 6 m. e) 5 m.
8. (ENEM-01) Um fabricante de brinquedos recebeu o projeto
de uma caixa que deverá conter cinco pequenos sólidos,
7. (UFF-2006) Populariza-se, na região da seca no nordeste do
colocados na caixa por uma abertura em sua tampa. A figura
Brasil, a construção de cisternas que armazenam as águas das
representa a planificação da caixa, com as medidas dadas em
chuvas. Uma vez tratada, a água abastecerá as famílias que ali
centímetros.
vivem.
(Texto adaptado de Discutindo Geografia. Ano 1 nº 3. 2005)

Considere os três recipientes a seguir que podem ser usados para


carregar água das cisternas.

O cilindro I tem a forma de um cilindro circular reto, de raio da


base igual a L e altura igual a 2L.

Os sólidos são fabricados nas formas de


I. um cone reto de altura 1 cm e raio da base 1,5 cm.
2L
II. um cubo de aresta 2 cm.
III. uma esfera de raio 1,5 cm.
IV. um paralelepípedo retangular reto, de dimensões 2 cm, 3cm e
L 4 cm.
V. um cilindro reto de altura 3 cm e raio da base 1 cm.

O fabricante não aceitou o projeto, pois percebeu que, pela


abertura dessa caixa, só poderia colocar os sólidos dos tipos
A) I, II e III B) I, II e V C) I, II, IV e V
51
PROF ANDRÉ FONSECA GEOMETRIA

D) II, III, IV e V E) III, IV e V

GABARITO
1-12 cm; 2-D; 3-A; 4-D; 5-50 ml; 6-E; 7-C; 8-C.

Questão Esfera

01. (UERJ-2012-2ºEQ)
Na fotografia abaixo, observam-se duas bolhas de
sabão unidas.

Quando duas bolhas unidas possuem o mesmo


tamanho, a parede de contato entre elas é plana,
conforme ilustra o esquema:

Considere duas bolhas de sabão esféricas, de mesmo


raio R, unidas de tal modo que a distância entre seus
centros A e B é igual ao raio R.
A parede de contato dessas bolhas é um círculo cuja
área tem a seguinte medida:

a)

b)

c)

d)

52

Você também pode gostar